Задачи на логику для взрослых ру с ответами: Загадки на логику для взрослых — сложные логические загадки с ответами

Содержание

Загадки на логику для взрослых — сложные логические загадки с ответами

В мифах Древней Греции есть история о храбром и мудром Эдипе, который правильно ответил на загадку страшного и непобедимого Сфинкса, тем самым сохранив свою жизнь. Острый ум и смекалистость всегда ценились в людях, вне зависимости от эпохи, в которой они родились и жили. Предлагаем раскрыть свой потенциал с помощью подборки интересных загадок, среди которых есть как смешные, так и сложные или с подвохом, поэтому будьте внимательны и не спешите сразу давать ответ.

Сложные логические загадки для взрослых

1. «Необычные цены»

Мужчина зашел в хозяйственный магазин и спросил цену на один товар. Продавец ему ответил:

  • один стоит 1 $;
  • восемь — 1 $;
  • тринадцать — 2 $;
  • сто шестьдесят восемь — 3 $;
  • четыре тысячи девятьсот восемьдесят шесть — 4 $.

Что покупал мужчина?

Показать ответ

Номерки на двери.

2.

«Свет»

Вы стоите перед закрытой комнатой. Внутри помещения есть 1 лампочка накаливания. Окна и щели, чтобы разглядеть свет внутри комнаты, отсутствуют.

Справа от двери ведущей в комнату установлено 3 переключателя, которые находятся в положении «выкл», но только один из них подключен к лампе. Нужно узнать, какой переключатель включает лампочку, но при этом зайти в комнату можно только 1 раз.

Примечание: менять положение переключателей разрешено неограниченное количество раз.

Показать ответ

Включите сразу два любых переключателя, а один оставьте в положении «выкл». Подождите 2 минуты, выключите один из включенных переключателей и сразу войдите в комнату:

  • если лампа горит, значит, это тот переключатель, который остался включенным;
  • если света в комнате нет, потрогайте лампочку: горячая — рабочий переключатель, который был выключен перед входом в помещение, холодная — подключен переключатель, положение которого ни разу не менялось.

3. «Строитель и кирпичи»

У строителя есть 8 кирпичей, 7 из которых имеют одинаковый вес, а один — тяжелее. Как строителю выявить бракованный кирпич с помощью весов всего за два взвешивания?

Показать ответ

Все нужно делать поэтапно:

  1. Разделить 8 кирпичей на три стопки по 3, 3 и 2 штуки.
  2. Взвесить 2 стопки, в которых по 3 кирпича.
  3. Если они уравновесились, значит взвесить стопку из двух кирпичей и найти бракованный.
  4. Если не уравновесились, то взять 3 кирпича из стопки, которая перевесила и разделить на 3 штуки.
    Положить по одному на весы. Если один перевесил — он бракованный, если уравновесились, значит, тяжелее всех тот, что остался в стороне.

4. «Блюдо для Фараона»

Повару древнего Египта необходимо варить яйцо в кипящей воде строго две минуты, но у него в наличии песочные часы только на три, четыре и пять минут. Как ему приготовить блюдо Фараону, не отклоняясь от времени, которое указано в рецепте?

Примечание: использовать часы можно на свое усмотрение: сразу все, только одни, по очереди или по несколько раз.

Показать ответ

Когда вода закипит, повару нужно поставить двое часов: на три минуты и на пять. Когда песок в трехминутных часах закончится, сразу опускать яйцо, таким образом, оно будет вариться требуемое время.

5. «Женский возраст — это тайна»

Парень спросил у девушки: «Сколько тебе лет?», на что она загадочно улыбнулась и ответила: «Позавчера мне было 22, а в следующем году будет 25».

Попробуйте догадаться, когда у девушки День рождения и какого числа происходил разговор между парой?

Показать ответ

Парень и девушка разговаривали 1 января, а День рождения был 31 декабря. Получается:

  • «позавчера» было 30 декабря и девушке было 22 года;
  • 31-го числа ей исполнилось 23;
  • в День рождения, который будет в тот год, когда велся разговор, исполнится 24;
  • а на следующий год девушка отметит 25-летие.

6. «Загадочный треугольник»

Каким образом нарисовать треугольник, чтобы все его углы были 90 градусов?

Показать ответ

На мяче или любом другом сферическом предмете.

7. «Мудрый отец»

Сын попросил у отца 10 000 долларов на обучение, на что тот ответил, что не может ему дать их просто так, поскольку не уверен в его способностях и в том, что он сможет закончить экономический факультет. Но предложил пари.

Если сын разложит 1000 долларов по 10 конвертам таким образом, чтобы какую бы сумму отец не назвал, парень мог дать ему это количество денег одним или несколькими конвертами, но при этом не доставая и не докладывая деньги внутрь, то он получит 10 000 долларов.

Сын пришел через 30 минут с 10 запечатанными конвертами и отец назвал сумму 555 долларов, на что парень ему выдал три конверта. Открыв, отец пересчитал все деньги и в сумме оказалось ровно 555 долларов, после чего выписал чек на 10 000 долларов ему на обучение. Каким образом сын разложил деньги по 10 конвертам, чтобы можно было получить любую сумму до 1000 долларов включительно?

Показать ответ

Парень разложил в конверты такие суммы в долларах: 1, 2, 4, 8, 16, 32, 64, 128, 256, 489.

8. «Простое добавление»

Если одиннадцать плюс два равняются одному, чему равны девять плюс пять?

Показать ответ

Речь идет о времени: 11 часов плюс 2 часа = 1 час, а 9 часов плюс 5 часов = 2 часа.

9. «Восемь восьмерок»

Запишите восемь восьмерок таким образом, чтобы в сумме получилась тысяча.

Показать ответ

888 + 88 + 8 + 8 + 8 = 1000.

10. «Две нити»

У вас есть две одинаковые по длине нити, и известно, что при зажигании одного конца любой из них горение будет длиться ровно один час. Как с помощью всего двух таких ниток и спичек отмерить 45 минут, не прикасаясь при этом руками к ниткам?

Показать ответ

Зажгите оба конца первой нити и один конец второй. Когда полностью сгорит первая нить, это будет сигнализировать о прошествии 30 минут. Сразу же подпалите второй конец горящей нитки таким образом, чтобы огонь шел навстречу друг другу. В результате, когда она полностью сгорит, пройдет 45 минут.

Смешные загадки на логику

Как проходит разминка? Все получилось разгадать? Уверены, что да! А теперь попробуйте потренироваться на более веселых и смешных загадках, которые также могут стать отличным развлечением среди друзей большой компании.

1. «Несмелый ковбой»

Ковбой вбегает в бар и кричит: “Срочно мне налейте стакан воды!” Вместо этого бармен выхватывает пистолет и стреляет в воздух. Ковбой испугался, поблагодарил бармена и вышел. За что ковбой остался благодарен бармену?

Показать ответ

У ковбоя была икота, и когда бармен выстрелил, от испуга она прошла.

2. «Злостный нарушитель»

У женщины не было при себе водительского удостоверения. Она проехала без остановки на железнодорожном переезде при опущенном шлагбауме. Потом, проигнорировав «кирпич» двинулась во встречном направлении по улице с односторонним движением и остановилась только через три перекрестка. Сотрудники ГИБДД, видя все это, решили не вмешиваться.

Почему?

Показать ответ

Женщина ехала на электричке, а потом шла пешком.

3. «Яркая профессия»

Олег зашел в магазин купить колбасы. Когда очередь дошла до него, он попросил порционно нарезать палку, при этом не поперек, а исключительно вдоль. Продавец поинтересовалась, не пожарником ли он случайно работает, на что Олег ответил: «Да». Как женщина догадалась о его профессии?

Показать ответ

Правильный ответ. Олег был в форме и вышел в обеденный перерыв.

4. «Практичные приметы»

Кому и при каких условиях не повезет встретить черную кошку «лицом к лицу»?

Показать ответ

Мыши. При любых условиях.

5. «Спортивный Санта»

Почему Санта Клаус так хорош в карате?

Показать ответ

Потому что у него черный пояс.

6. «Милые мишки»

Как называется медведь, у которого нет зубов?

Показать ответ

Мармеладный мишка.

7. «Слезы змеи»

Как заставить расплакаться гремучую змею?

Показать ответ

Забрать у нее погремушку.

8. «Конфуз в блошином цирке»

Почему собаку выгнали из блошиного цирка?

Показать ответ

Пес украл всех артистов шоу!

9. «Неожиданная поза»

Что происходит с курицей, когда она лежит вверх ногами?

Показать ответ

Ее приготовили и едят.

10. «Странная схожесть»

Что общего между ежиком и молоком?

Показать ответ

Они оба могут сворачиваться.

Логические загадки для взрослых с подвохом

А теперь попробуйте мыслить нестандартно и креативно, стараясь заметить мелочи и не повестись на уловки, ведь далее представлена подборка загадок с подвохом.

1. «Непонятный возраст»

Одной женщине в 1850 году исполнилось 40 лет, но в 1860 году ей было 30 лет. Как такое могло получиться?

Показать ответ

В 1850 году женщине только исполнилось 40 лет, в то же время в 1860 году ей уже было 30, получается 1860 год был раньше чем 1850, а такое возможно только в том случае, если летоисчисление ведется до рождества Христова, то есть 1850 год до н.

э. и 1860 год до н. э.

2. «Игра до рассвета»

Четверо мужчин сели играть. Игра длилась всю ночь до самого рассвета. Причем все они играли за деньги, а не ради удовольствия. Когда пришло время сводить счета, мужчины заработали приличные суммы и ни один из них не проиграл.

Как такое могло выйти?

Показать ответ

Эти мужчины — члены оркестра, нанятого для частной вечеринки.

3. «Правила дорожного движения»

Перед перекрестком на светофоре стоят и ждут зеленого света: фура, телега с лошадью и мотоциклист. Загорелся желтый, водитель фуры нажал на газ, отчего лошадь испугалась и укусила за ухо мотоциклиста. Кто из троих нарушил ПДД?

Показать ответ

Мотоциклист, так как был без шлема.

4. «Человек дождя»

Художник гулял по парку, когда на улице начался дождь. С собой у мужчины не оказалось шляпы и зонтика, а в кронах деревьев из-за сильного ливня укрыться не вышло. В результате вся одежда оказалась влажной, но ни один волос на голове художника не промок. При каких обстоятельствах такое могло произойти?

Показать ответ

Это был лысый художник.

5. «Провинившийся провидец»

Находясь в лагере, развернутом на поле во время военных действий, солдат был в ночном карауле. Около 4-х часов утра он разбудил командующего и сообщил, что этой ночью увидел во сне, как противники атакуют их лагерь с юга. И хотя командующий не был суеверным человеком, все же выставил наблюдателей за южной частью равнины.

Спустя некоторое время, противник напал именно с указанной солдатом стороны, но так как меры были предприняты заранее, войско смогло с легкостью отбить атаку и напасть в ответ на противника, победив его. После изнурительного боя и одержанной победы командующий поблагодарил солдата и посадил его в тюрьму. Почему?

Показать ответ

Сны человеку снятся, только когда он спит, а караульному это запрещено.

6. «Смекалистый отличник»

Перед началом экзамена профессор объявил студентам: «На этом письменном экзамене есть три вопроса. У вас будет всего один час, чтобы ответить на все, и не более. Любой, кто будет писать дольше, получит двойку без права на пересдачу!».

Андрей, получив свой билет, принялся отвечать. После того как пропищал таймер, осталось всего 5 минут, студент понял, что ответил только на два вопроса и часть третьего. Осмотревшись в аудитории, парень заметил, что с их группой сдает другой экзамен старший курс, а значит, ему может сойти с рук несколько лишних минут, пока другие будут сдавать работы.

Закончив отвечать на вопрос, Андрей пошел сдавать работу, но по дороге его перехватил профессор и сказал: «Я видел, как после завершения одного часа, ты продолжал нагло писать, а потому заслуживаешь только двойку!»

Парень подумал и спросил: «Вы знаете, кто я?» На что профессор покачал головой, подтвердив, что даже не имеет представления, кто он такой, и ответил: «Меня не волнует, кто ты такой. А вот тебе нужно уважать меня и соблюдать дисциплину!»

– Хорошо! — ответил Андрей, положил работу и ушел. В день объявления результатов он получил оценку «отлично». Почему?

Показать ответ

Когда профессор подтвердил, что не знает, кто такой Андрей, тот быстро сунул экзаменационную работу в общую стопку на столе и ушел, чтобы профессор не знал, какая работа именно его и заслуживает неудовлетворительной оценки за несвоевременную сдачу. Поэтому во время проверки это требование профессор просто не учел и оценивал лишь знания студента, которые показали отличный результат!

7. «Знаменитости»

Наполеон Бонапарт, Петр Первый, Юрий Гагарин, Шерлок Холмс, Вильям Шекспир, Нострадамус, Пифагор. Кто лишний в этом списке?

Показать ответ

Шерлок Холмс, поскольку это вымышленный персонаж, а остальные — реальные люди.

8. «Восемь подружек»

Восемь подружек находятся в доме и каждая из них занята каким-то делом:

  • первая — готовит ужин;
  • вторая — разжигает камин;
  • третья — читает книгу;
  • четвертая — играет в шахматы;
  • пятая — рисует картину;
  • шестая — убирается в доме;
  • седьмая — вешает шторы.

Чем занята восьмая девочка?

Показать ответ

Играет в шахматы вместе с четвертой подружкой.

9. «Прогноз погоды»

Если в 12 часов ночи пошел снег, можно ли предположить, что через 96 часов на улице будет солнце?

Показать ответ

Такого быть не может так как 96 часов — это ровно четверо суток, а значит, через указанный промежуток времени снова будет 12 часов ночи.

10. «Четыре сестры»

У мамы Мэри четыре дочери. Имя первой — Априна, второй — Мая, третьей — Июния.

Как зовут четвертую девочку?

Показать ответ

Ее зовут Мэри.

Математика и логика для детей 7-13 лет

Развиваем логическое мышление через решение сюжетных математических задач в интерактивном игровом формате

узнать подробнее

Сложные загадки на логику — логические задачи с ответами

Логическая загадка любой сложности заставит вас подумать о давно забытых предметах, вспомнить содержание детских сказок и смириться с тем, что ваш ребенок находит ответы быстрее вас. Хотите проверить? Мы подготовили специальную подборку сложных логических загадок!

Зачем решать логические загадки?

При решении логических задачек и поиске «отгадки» у человека начинают работать оба полушария мозга. Левое полушарие отвечает за логику и стремится разобраться в причинных связях. Правое несет ответственность за интуицию, учится строить целостную картину и формировать образное мышление.

У современных людей левое полушарие включается в работу не так часто — «загуглили» и отправились по делам. И, конечно, свою “логическую мышцу” можно и нужно тренировать — тут нам и пригодятся загадки на логику – они помогают держать мозг в тонусе даже в самые «ленивые» дни.

Эффект от «логических тренировок» даст о себе знать уже после первых занятий — натренированный мозг быстрее соображает, позволяя решать не только математические задачи, но и находить выход из жизненных ситуаций.

Зачем детям развивать логику?

Развитое логическое мышление поможет вашему ребенку легко справляться с математическими задачами не только в начальной, но и в старшей школе.

Умение анализировать выручит его на литературе, а способность нестандартно мыслить пригодится в творческих кружках и школьных активах. И, конечно, он всегда сможет дать аргументированный ответ на поставленный учителем каверзный вопрос, а еще переспорит одноклассников и точно завоюет авторитет в школе!

Боитесь, что ваш ребенок не справится, потому что «гуманитарий»? В Умназии вы найдете сотни загадок на логику, которые адаптированы для детей разных классов и возрастов, которые позволяют развивать Логику постепенно.

Не отказывайтесь от «мозговых тренировок» — начните мыслить не так, как все!

Сложные загадки на логику

Давайте попробуем разгадать 5 загадок на логику. Дети и взрослые могут размышлять над ответом вместе.

Загадка №1

Каких камней не бывает в речке?

Показать ответ

Ответ: В речке вы никогда не найдете сухих камней. А вот драгоценные попасться могут 🙂

Загадка №2

На столе лежат две монеты, в сумме они дают 3 рубля. Одна из них — не 1 рубль. Какие это монеты?

Показать ответ

Ответ: На столе лежат 2 рубля и 1 рубль. В условии сказано, что только одна из монет – не рубль.

Загадка №3

Что не вместится даже в самую большую кастрюлю?

Показать ответ

Ответ: Крышка этой кастрюли 🙂 Не ходите на кухню и не проверяйте – точно застрянет!

Загадка №4

Что может в одно и то же время стоять и ходить, висеть и стоять, ходить и лежать?

Показать ответ

Загадка №5

Завязать можно, а развязать нельзя. Что это такое?

Показать ответ

Ответ: Нет, не шнурки от старых кроссовок. Правильный ответ — разговор.


Справились? Если да – вы молодцы. А если что-то не получилось – приходите в Умназию и тренируйте ум с умом!

Умназия – образовательная онлайн-платформа для учеников начальной школы. На платформе ребенок сможет:
Развить логику и внимание на тренажере навыков, прокачать память и стать финансово грамотным!

У нас ребенку не будет скучно: умные алгоритмы подготовят для него индивидуальную программу, а в онлайн-тренажере он будет раскрывать тайны, получать достижения и становиться героем Умназии.

А для самых умных и мотивированных ребят у нас проводятся олимпиады по 4 предметам, которые готовят детей к будущим олимпиадам в школах. Приходите и развивайтесь вместе с нами!


Теперь приступим к более сложным заданиям. Включайте логику и начинайте!

Загадка №6

Я – вода, и по воде плаваю. Кто я такая?

Показать ответ

Ответ: Льдина.

Загадка №7

Ползут 3 черепахи.1-я черепаха говорит: за мной ползут две черепахи.
2-я черепаха говорит: за мной ползёт одна черепаха и передо мной ползёт одна черепаха. А 3-я черепаха: передо мной ползут две черепахи, и за мной ползёт одна черепаха.

Как такое может быть?

Показать ответ

Ответ: Черепахи ползут по кругу!

Загадка №8

На ферме было 2 коня, 1 кролик, 1 щенок, 1 кошка, свинья и поросенок, корова и теленок, индюк и гусь.
Пришел хозяин с собакой. Сколько на ферме стало ног?

Показать ответ

Ответ: 26. Почему? Потому что ноги только у человека, лошадей, свиней и коров. У остальных животных — лапы.

Загадка №9

Где впервые был обнаружен картофель?

Показать ответ

Ответ: Отгадка предельно простая – в земле.

Загадка №10

Странный дождь порой идет: сотней струй он кверху бьет.

Показать ответ

Ответ: Фонтан.


Почему логические загадки полезны для детей? Они развивают смекалку, учат работать с информацией, «прокачивают» логико-математический интеллект и делают ребенка более самостоятельным.

Помимо успехов в школе, о которых мы уже говорили, развитое логическое мышление способствует формированию уверенности в себе в процессе повседневной жизни.

>25 тысяч учеников уже решают авторские задачи в Умназии!


А теперь продолжим тренироваться, ведь долго без логики не протянешь!

Загадка №11

Скажешь «не приходи!» — всё равно приходит. Скажешь «не уходи!» — всё равно уходит. Что это такое?

Показать ответ

Загадка №12

Вы сидите в самолете, впереди вас лошадь, сзади автомобиль. Где вы находитесь?

Показать ответ

Ответ: Вы катаетесь на карусели.

Загадка №13

Чем больше из нее берешь, тем больше она становится. Что это?

Показать ответ

Загадка №14

Что принадлежит вам, однако другие этим пользуются чаще, чем вы сами?

Показать ответ

Ответ: Ваше имя.

Загадка №15

Сколько яиц можно съесть натощак?

Показать ответ

Ответ: нет, размер вашего аппетита тут не при чем. Натощак можно съестьскушать только 1 яйцо, потому что все следующие будут съедены уже не на пустой желудок.

На сегодня это все — мы с вами славно потрудились.

Не хотите останавливаться на достигнутом? Регистрируйтесь в на нашей платформе и решайте тысячи задач в онлайн-режиме. Создавайте индивидуальный учебный план для вашего ребенка и развивайте пять навыков в удобное время.

Начните заниматься с ребенком уже сегодня!

Логика и математика для детей 7-13 лет

Развиваем логическое мышление, учим работать с информацией и принимать верные решения

узнать подробнее

Читайте также:


 

Загадки на логику для взрослых: отборные загадки с ответами

Искали просто 10-20 загадок для взрослой компании? Продолжайте читать!

Простые и сложные, серьезные и забавные логические загадки на смекалку и с подвохом помогут поднять настроение и отлично провести время любой взрослой компании.

Даже если с вами будут дети, им не придется закрывать уши. Подход LogicLike — проводите время весело и задорно всей семьей!

Серьезные логические загадки

Не лёд, а тает, не лодка, а уплывает.

Эти три телезвезды давно на экране. Одного зовут Степан, второго — Филипп. Как зовут третьего?

Что нужно делать, когда видишь зеленого человечка?

– Она красная?
– Нет, черная.
– А почему она сейчас белая?
– Потому, что еще зеленая.
О чем речь?

Логические загадки с подвохом с ответами для детей

Логические задачи с подвохом — это нестандартные задачи. На первый взгляд в вопросах нет ничего необычного. Но задачи с подвохом, как правило, не без чувства юмора. Чтобы дать правильный ответ нужно быть очень внимательным и уметь размышлять. Задачи с подвохом отлично развивают сообразительность и нестандартное мышление. Они побуждают ребёнка прислушиваться к речи: к интонации, фонетическим нюансам, распознавать многозначность одной и той же фразы.

Например, на вопрос «Почему гуси плавают?» сразу хочется ответить: «Ну уж такими их создала природа». Но на самом деле надо услышать не «Почему», а «По чему», и тогда ответ будет: «По воде». Вопрос «Что в начале книги?» не надо понимать буквально. Правильный ответ: «Буква К».

Задания непростые. Несколько раз, медленно прочтите ребёнку вопрос. Дайте ему время подумать и найти верный вариант.

Начнем отгадывать с прорисованных задач. Это несколько страничек из моей книги.

Ответ

Буква Ц. Орехи не умеют ходить.

 

Ответ

Два: правый и левый. Дождаться, когда она улетит.

Ответ

Буква К. Нет, пингвины не разговаривают.

 

А теперь пусть ребенок все представляет в своей голове. Это тоже очень полезно, особенно для развития наглядно-образного мышления.

  1. Что может быть больше слона и одновременно невесомым?
  2. Что у коровы позади, а у аиста впереди?

    Ответ

    (буква а)

  3. Сколько горошин войдёт в стакан?

    Ответ

    (горошины не ходят)

  4. Почему, когда хочешь спать, идёшь в кровать?

    Ответ

    («по чему» — по полу)

  5. Что бросают в кастрюлю, прежде чем в неё что-то положить?
  6. Какой конь не ест овса?

    Ответ

    (шахматный)

  7. Что может в одно и тоже время стоять и ходить, висеть и лежать, ходить и бежать?
  8. Как написать «сухая трава» четырьмя буквами?
  9. Почему цапля стоит на одной ноге?

    Ответ

    (если цапля поднимет вторую ногу, то упадёт)

  10. Каким гребнем волосы не расчешешь?

    Ответ

    (петушиным)

  11. Что не войдёт даже в самую большую кастрюлю?

    Ответ

    (её крышка)

  12. Почему курица кладёт яйца?

    Ответ

    (потому что, если она их бросит, то они разобьются)

  13. Когда котёнка покупают, каким он бывает?

    Ответ

    (мокрым)

  14. Что поднять с земли легко, но трудно кинуть далеко?
  15. На что похожа половина яблока?

    Ответ

     (на вторую половину)

  16. Чем кончается лето и начинается осень?

    Ответ

    (буквой О)

  17. Из какого крана не берут воду?

    Ответ

    (из подъёмного крана)

  18. Какой колокольчик не звенит?

    Ответ

    (цветок)

  19. Может ли страус назвать себя птицей?

    Ответ

    (нет, потому что страусы не умеют говорить)

  20. Какой рукой лучше размешивать чай?

    Ответ

    (в которой ложка)

  21. На какой вопрос не дашь утвердительный ответ?

    Ответ

    («Ты спишь?»)

  22. Какая птица яиц не несёт, но сама из них вылупляется?
  23. Что идёт из города в город, но при этом не движется?

    Ответ

    (дорога)

  24. Что можно увидеть с закрытыми глазами?
  25. Сколько поворотов у дороги?

    Ответ

    (два-правый и левый)

  26. Как можно снять колесо за одну секунду?

    Ответ

    (фотоаппаратом)

  27. Каких камней нет в море?
  28. Что находится в середине капусты?

    Ответ

    (буква У)

  29. Какая птица состоит из буквы и реки?

    Ответ

    (и-Волга)

  30. Зачем мы едим?

    Ответ

    (за столом)

  31. Кто ходит сидя?

    Ответ

    (шахматист)

     
  32. На какое дерево садится птица во время проливного дождя?

    Ответ

    (на мокрое)

     
  33. Когда забивают гвозди, почему стучат молотком?

    Ответ

    (по шляпкам)

  34. Как написать «мышеловка» пятью буквами?
  35. Когда машина едет, какое колесо не крутится?

    Ответ

    (запасное)

  36. Отчего утка плавает?

    Ответ

    (от берега)

  37. Несла бабка на базар сто яиц, одно упало. Сколько яиц осталось в корзине?

    Ответ

    (ни одного, потому что дно упало)

Размышляя над логическими задачами с подвохом, ребёнок поймёт, что за одними и теми же словами часто скрыто гораздо больше, чем можно услышать с первого раза. Это отличная тренировка мышления и внимания.

На сайте можно также бесплатно скачать книгу «Логические коврики», которые отлично тренируют навык работы с закономерностями.

С уважением, Ольга Наумова

 

Благодарю, что поделились статьей в социальных сетях!

Вопросы-головоломки на собеседованиях с ответами. Часть 1 — Work.ua

Логические задачи кандидатам на вакансию задают, чтобы оценить их аналитические способности. С подобными вопросами может столкнуться каждый. Но даже, если с вами это не случится — изучить их для подготовки будет просто интересно.

Соискателям, столкнувшимся с головоломками в процессе собеседования, не стоит паниковать и теряться. Как правило, в большинстве случаев достаточно показать ход ваших мыслей и пути решения поставленной задачи. Вполне может быть, что у задачи и нет правильного решения или однозначного ответа. Ни в коем случае нельзя говорить «я не знаю» или «затрудняюсь ответить», просто размышляйте вслух.

Чтобы вы знали к чему быть готовыми и не растерялись, Work.ua собрал самые популярные вопросы-головоломки и подготовил ответы и рекомендации к ним. Кто знает, может вам попадутся именно эти задачи.

Все эти головоломки могут вам задать при собеседовании на вакансию программиста php.

Канализационные люки

Это, пожалуй, самый распространенный вопрос среди всех головоломок.

Вопрос: Почему канализационные люки круглые?

Ответ: Есть несколько вариантов. Так как диаметр круга одинаков, как его не крути, то круглый люк не может провалиться в колодец. У квадрата же, например, диагональ больше его сторон, поэтому крышка могла бы упасть. Также можно ответить, что круглые люки проще транспортировать и передвигать.

Лампочки

Вопрос: В закрытой комнате есть 3 лампочки, а в коридоре 3 выключателя. За какое минимальное открытие дверей можно определить какой выключатель к какой лампочке относится?

Ответ: За одно открытие. Включаем одновременно 2 выключателя, через некоторое время выключаем второй. Заходим в комнату: одна из лампочек осталась гореть — это первый выключатель, далее щупаем оставшиеся лампочки — теплая и будет вторым выключателем, а холодная соответственно третьим.

Торт

Вопрос: Как разделить торт на 8 равных частей тремя разрезами?

Ответ: Сперва нужно сделать 2 разреза крест на крест, поделив торт на 4 равных части. А затем разрезать торт горизонтально пополам. Ну и что, что куски стали невысокие, зато у вас 8 равных частей. Еще можно после первых двух разрезов сложить кусочки друг на друга и переполовинить одним разрезом.

Мертвый человек и спичка

Вопрос: Среди поля найден мертвый человек со спичкой в руках, следов нет. От чего он умер и при каких обстоятельствах?

Ответ: Человек умер от падения с самолета, который начал терять высоту, и авария была неизбежна. На всех пассажиров не хватило одного парашюта и они тянули жребий. Ему досталась короткая спичка, и он был вынужден прыгать без парашюта.

Автобус и мячи

Вопрос: Сколько теннисных мячей поместится в автобус?

Ответ: Точного правильного ответа не знает и сам рекрутер, так как не уточняется, что за мячи и автобус — их размеры не известны, и никто досконально это не проверял. Поэтому тут важен ход ваших мыслей, вы можете только предположить. Назовите примерные длину, ширину и высоту автобуса, размеры одного мяча. Посчитайте объем автобуса и мяча — так вы узнаете сколько мячей поместится в пустой автобус. Уменьшите примерно это значение с учетом сидений и других деталей автобуса, сделайте поправку на то, что мячи не квадратные и дайте ответ. В этом случае процесс поиска ответа важнее самого ответа. Варианты вопроса могут быть разные: мячи — футбольные, вместо автобуса — комната и т.п.

Таблетки

Вопрос: Доктор выдал пациенту 4 таблетки двух видов — по 2 таблетки каждого, которые нельзя отличить по внешнему виду. Таблетки надо выпить за два приема: утром по одной таблетке каждого вида и так же вечером. Если нарушить дозировку или не принять таблетки, то пациент умрет. Так вышло, что таблетки перемешались. Как пройти курс лечения и выжить?

Ответ: Конечно, можно сказать, что лучше пойти к врачу и попросить еще, все таки вопрос жизни и смерти. Но это могут быть единственные таблетки на Земле, доктор может исчезнуть при загадочных обстоятельствах и т.д. Так что ответить все же придется. К тому же все достаточно просто: нужно разделить каждую таблетку на 2 части и выпить по половинке каждой таблетки утром и вечером.

В заключение

Помните, что прежде всего такие задачи и головоломки предназначены для того, чтобы проверить поведение кандидата в нестандартных ситуациях, оценить способность к размышлению, творческому и логическому подходу. Увы, нередки случаи, когда интервьюер не может правильно интерпретировать результаты таких вопросов, или вовсе не понимает их предназначения. Но даже в этой ситуации уверенное поведение и стремление прийти к ответу покажут вас с лучшей стороны и увеличат шансы получить должность.


Читайте также: Вопросы-головоломки на собеседованиях с ответами. Часть 2



Чтобы оставить комментарий, нужно войти.

Логика 7 8 лет

Логика (7–8 лет)

Вспомним знакомую с детства сценку. Мальвина говорит Буратино: «У вас в кармане два яблока». «Врете, ни одного…» — отвечает озорной деревянный мальчишка. Эта сценка в полной мере отражает возрастную особенность детского мышления, а именно конкретность. Свойство детского ума воспринимать все конкретно, буквально, неумение подняться над ситуацией и понять ее общий смысл — одна из основных трудностей детского мышления, ярко проявляющаяся при изучении таких абстрактных школьных дисциплин, как математика или грамматика. Однако к моменту поступления в первый класс образное мышление детей перестает быть сугубо конкретным и ситуативным. Ребенок способен не только представить предмет во всей полноте и разнообразии характеристик, но также способен выделить его существенные свойства и отношения. У него формируется наглядно-схематическое мышление. Это особый вид мышления, который выражается в том, что ребенок понимает и успешно использует различные схематические изображения предмета (план, макет, простейший чертеж). Дети начинают понимать и условные изображения значительно более абстрактных взаимосвязей: отношений между словами в предложении, между буквами в слове, между математическими величинами и т.д. Это открывает путь к обучению детей грамоте и математике с опорой на наглядно-условные отображения основных закономерностей внутри учебного материала. Начинают закладываться основы словесно-логического мышления. Этот вид мышления окончательно формируется только в подростковом возрасте (13–14 лет) и является ведущим у взрослого человека.  

Процесс отвлечения в возрасте 7–8 лет совершается не только при восприятии ряда предметов, но и под влиянием словесных описаний и объяснений. Однако ребенок все еще находится в плену образов конкретных предметов. Зная из опыта, что железные предметы тонут в воде, он говорит, что гвоздь утонет, но этот вывод он подкрепляет не общим положением («Все железные предметы тонут»), а ссылкой на единичный случай: «Я сам видел, как гвоздь тонул».

Об активности мышления детей красноречиво говорят их многочисленные вопросы, в которых ребенок выражает любознательность к тому, что его окружает: «Почему теперь ночь? Почему капель падает? Отчего огонь в спичке, где он спрятан?» и т.д. Мысль почемучки теперь направлена на различение и обобщение предметов, явлений, событий, наблюдаемых ими.

В протекании мыслительного процесса также выступает наметившийся еще у пяти-, шестилетних детей способ мышления методом «короткого замыкания». Ребенок не анализирует всю задачу в целом (житейскую, орфографическую или математическую), т. е. не выделяет всех ее условий, всех данных и не видит связи между ними. Он выхватывает какое-то одно условие и устанавливает прямую связь с любым другим условием или с заданным вопросом. Так, отгадывая загадку «Я все знаю, всех учу, но сама всегда молчу», первоклассник часто отвечает, что это учительница. Фразы «Я все знаю, всех учу…» достаточно, чтобы найти решение, т.е. подставить знакомый образ. И хотя дальше следует дополнение — «…но сама всегда молчу», — т.е. налицо как будто прямо противоположное условие найденной отгадке, ребенок просто отбрасывает это условие.

Поступление в школу меняет содержание деятельности детей. Значительно расширяется круг предметов и явлений, о которых они должны думать, повышаются и требования к самим процессам мышления. Преподаватель учит детей внимательно следить за ходом рассуждения, точно выражать мысли в словах, сначала думать, а потом делать что-либо и пр. Хотя мышление младших школьников в целом остается еще конкретно-образным, элементы абстрактного мышления выражены все заметнее. Дети могут мыслить на уровне общих понятий о том, что им хорошо известно, о знакомых животных, растениях, людях и их труде.

Темп развития мышления детей школьного возраста в значительной мере зависит от того, как их обучают. Опытное обучение младших школьников по специальным программам повышенной трудности доказывает, что уже у детей 7–8 лет способность к отвлеченному рассуждению и последовательному выполнению умственных действий достаточно высока. Применение научно разработанных методов обучения детей убыстряет развитие мышления. 

Таким образом, в формировании мышления школьников решающее значение принадлежит учебной деятельности, постепенное усложнение которой ведет за собой развитие умственных способностей учащихся.

Однако для активизации и развития мыслительной деятельности детей бывает целесообразно использовать неучебные задания, которые в целом ряде случаев оказываются для школьников более привлекательными. Неоценимую помощь в развитии логического мышления окажут задания и упражнения на поиск закономерностей, логические задачи, головоломки. Пусть ребенок отгадывает загадки и придумывает их сам. Знакомьте его с пословицами, но не в абстрактной форме, а применительно к жизненной ситуации (например, если ребенок разбросал игрушки, скажите: «Любишь кататься — люби и саночки возить», и объясните обобщенный смысл пословицы).

Сделайте занятия по развитию мышления ребенка не только полезными, но и увлекательными. В этом вам обязательно поможет наш сайт!

Удачи и гордости за достижения вашего ребенка!

100 вопросов на логику. Логические и занимательные задачи (300 задач)

Веселые вопросы для разминки ума

1. Из пункта А в пункт Б выехал поезд. Навстречу ему одновременно из пункта Б в пункт А выехал другой поезд. Дорога одноколейная, но они не встретились. Почему?
Ответ: Не судьба.

2. Столовый прибор для хлебания щей?
Ответ: Лапоть.

3. Если за столом сидят шесть евреев, что находится под столом?
Ответ: Двенадцать колен Израилевых.

4. Почему человек назад оглядывается?
Ответ: Потому, что у него глаз на затылке нет.

5. Какой рукой лучше размешивать чай?
Ответ: Чай лучше размешивать ложкой.

6. С какой скоростью должна бежать собака, чтобы не слышать звона сковородки, привязанной к ее хвосту?
Ответ: Собаке достаточно стоять на месте.

7. Когда вы смотрите на змею, как узнать, самка это или самец?
Ответ: Если змея поползла, то это самка, а если пополз, то самец.

8. Можно ли прыгнуть выше девятиэтажки?
Ответ: Да, девятиэтажка не умеет прыгать.

9. Знаете ли вы, что такое плевательница для бездельников?
Ответ: Потолок.

10. Как правильно говорить: «Я не вижу белый желток» или «Я не вижу белого желтка»?
Ответ: Вообще-то желток желтого цвета.

11. До каких пор можно зайти в лес?
Ответ: До середины, потом уже будешь выходить.

Где находится край света… и другие забавные вопросы

1. Где находится край света?
Ответ: Там, где начинается тень.

2. На что люди обувь покупают?
Ответ: На деньги.

3. Может ли дождь идти два дня подряд?
Ответ: Не может, потому что ночь разделяет дни.

4. Среднее арифметическое между велосипедом и мотоциклом?
Ответ: Мопед.

5. Может ли мужчина женится на сестре своей вдовы?
Ответ: Нет, вдова — это жена умершего мужа.

6. Из какой посуды нельзя ничего поесть?
Ответ: Из пустой

7. По чему часто ходят и никогда не ездят?
Ответ: По лестнице.

8. Из какого полотна нельзя сшить рубашку?
Ответ: Из железнодорожного

9. На что похожа половина яблока?
Ответ: На вторую половину.

10. Какой полуостров жалуется на свою величину?
Ответ: Ямал.

11. Без чего не испечь хлеб?
Ответ: Без корки.

12. На какое дерево садится ворона во время дождя?
Ответ: На мокрое.

13. На каких полях трава не растет?
Ответ: На полях шляпы.

14. Каких камней нет в море?
Ответ: Сухих.

15. Какой узел нельзя развязать?
Ответ: Железнодорожный.

16. Какая река самая хищная?
Ответ: Тигр.

17. Какой месяц короче всех?
Ответ: Май (в его названии всего три буквы).

18. Без чего человеку жить нельзя?
Ответ: Без имени.

19. Каким гребнем голову не расчешешь?
Ответ: Петушиным.

20. Его можно дать, и в то же время сдержать, что это?
Ответ: Слово.

Хитрые вопросы на логику и сообразительность

1. Какое слово всегда звучит неверно?
Ответ: Слово «неверно».

2. На столе лежат линейка, карандаш, циркуль и резинка. На листе бумаги нужно начертить окружность. С чего Вы начнете?
Ответ: Надо достать лист бумаги.

3. У трех родственных трактористов есть брат Сергей, у Сергея братьев нет. Может ли такое быть?
Ответ: да, если трактористы — женщины.

4. Шли два отца и два сына. Нашли три апельсина. Не резали, не пилили, а поровну разделили. Как это может быть?
Ответ: это были дед, отец и сын.

5. Назовите пять дней, не называя чисел и названий.
Ответ: Позавчера, вчера, сегодня, завтра, послезавтра.

6. Кто может путешествовать по свету, оставаясь в одном и том же углу?
Ответ: Почтовая марка на конверте.

7. Как спрыгнуть с десятиметровой лестницы и не разбиться?
Ответ: Спрыгнуть с первой ступеньки.

8. На какой вопрос нельзя ответить «нет»?
Ответ : Вы живы?

9. Из гнезда вылетели три ласточки. Какова вероятность того, что через 15 секунд они будут находиться в одной плоскости?
Ответ: Три точки всегда образуют одну плоскость.

Дошкольники решают эту задачу за 5-10 минут. У некоторых программистов уходит на неё до часа. Но многие люди, исписав несколько листов бумаги, сдаются.

Номер парковочного места

На решение этой задачи у шестилетнего ребенка уходит обычно не больше 20 секунд. А вот неподготовленных взрослых она часто вводит в ступор. Так какое же число скрыто под машиной?

Загадка для гения

Гений находит решение за 10 секунд. Билл Гейтс — за 20 секунд. Выпускник Гарварда (Harvard University) — за 40 секунд. Если вы нашли ответ за 2 минуты, то вы принадлежите к 15% наиболее одаренных людей. 75% людей не способны решить эту задачу.

Правитель острова

Самодержавный правитель одного острова хотел воспрепятствовать тому, чтобы на острове поселились пришельцы. Желая соблюсти видимость справедливости, он издал распоряжение, согласно которому всякий, желающий поселиться на острове должен, хорошо поразмыслив, высказать любое утверждение, причем после предварительного предупреждения, что от содержания этого утверждения зависит его жизнь. Распоряжение гласило: «Если пришелец скажет правду, его расстреляют. Если он скажет неправду, его повесят». Может ли пришелец стать жителем острова?

Утверждение проекта

Согласно договоренности, порядок утверждения нового проекта, в разработке которого участвуют учреждения А, Б, В, таков: если в утверждении принимают сначала участие А и Б, то должно подключиться к участию и учреждение В. Если утверждение происходит сначала в учреждениях Б и В, присоединяется и учреждение А. Спрашивается: возможны ли такие случаи при утверждении проекта, когда принимали бы в нем участие только учреждения А и В, между тем, как участие учреждения Б не было бы необходимо (при сохранении договоренности о порядке утверждения проектов)?

Два племени

На острове живут два племени: молодцы. Которые всегда говорят правду, и лжецы, которые всегда лгут. Путешественник встретил островитянина, спросил его, кто он такой, и когда услышал, что он из племени молодцов, нанял его в проводники. Они пошли и увидели вдали другого островитянина, и путешественник послал своего проводника спросить его, к какому племени он принадлежит. Проводник вернулся и сказал, что тот утверждает, что он из племени молодцов. Спрашивается: был проводник молодцом или лгуном?

Аборигены и пришельцы

Перед судом стоят три человека, из которых каждый может быть либо аборигеном, либо пришельцем. Судья знает, что аборигены всегда отвечают на вопросы правдиво, а пришельцы всегда лгут. Однако судья не знает, кто из них абориген, а кто — пришелец. Он спрашивает первого, но не понимает его ответа. Поэтому он спрашивает сначала второго, а потом третьего о том, что ответил первый. Второй говорит, что первый говорил, что он абориген. Третий говорит, что первый назвал себя пришельцем. Кем были второй и третий подсудимые?

Жук на ленте

Жук отправился в путешествие. Он ползет по ленте, длина которой 90 сантиметров. На другом конце ленты, в двух сантиметрах от конца, — цветок. Сколько сантиметров придется ползти жуку до цветка: 88 или 92 (при условии, что ползает он все время по одной стороне и лишь в конце может через торец ленты перебраться на другую сторону)?

Покупка

Марина долго выбирала, какой кувшинчик купить. Наконец выбрала. Продавщица уложила покупку в коробку. Что купила Марина? Сколько кувшинов продавщица поставила на полки, на каких они стояли раньше?

Турист

Турист шел к озеру. Он дошел до перекрестка, откуда вела одна дорога направо, а другая – налево; одна шла к озеру, другая – нет. На перекрестке сидели двое парней, один из них всегда говорил правду, второй всегда лгал. Оба они отвечали на любой вопрос либо «да», либо «нет». Все это было туристу известно, но он не знал, кто из них говорит правду, а кто лжет; он также не знал, какая из дорог ведет к озеру. Турист поставил лишь один вопрос одному из парней. Какой это был вопрос, раз он узнал по ответу, какая дорога ведет к озеру?

Разбитое окно

В перерыве в классе оставалось девять учеников. Один из них разбил окно. На вопрос учителя были получены следующие ответы:

Сколько треугольников? Какая команда?

Читайте внимательно и ничего не записывайте: «Торпедо» возглавляет турнирную таблицу, «Спартак» находится на пятом месте, а «Динамо» как раз посередине между ними. Если «Локомотив» опережает «Спартака», а «Зенит» занимает место сразу же за «Динамо», то какая из перечисленных команд находится на втором месте? На раздумье дается 30 секунд.

Порядок утверждения проектов

На предприятии есть три цеха – A, B, C, договорившиеся о порядке утверждения проектов, а именно: 1. Если цех B не участвует в утверждении проекта, то в этом утверждении не участвует и цех A. 2. Если цех B принимает участие в утверждении проекта, то в нем принимают участие цехи A и C. Спрашивается: обязан ли при этих условиях цех C принимать участие в утверждении проекта, когда в утверждении принимает участие цех A?

Вечерняя прогулка

Кто из этих девяти усачей отправился на «вечернюю прогулку»?

7 кнопок

Какую из 7 кнопок надо нажать. Чтобы звонок зазвонил? Рекомендуется найти путь мысленно.

Составьте таблицу

В московском полуфинале первенства Европы по баскетболу, проходившем в советское время, места распределились следующим образом: СССР – 14 очков, Италия и Чехословакия – по 12, Израиль – 11, Финляндия – 10, ГДР и Румыния – по 9 и Венгрия – 7 очков. Согласно положению. Каждая команда за выигрыш получала 2 очка, за поражение – 1 очко, за неявку – 0 очков. Ничьи не допускались. Составьте сводную таблицу результатов игр, если известно, что команда Финляндии выиграла у команды Италии и проиграла команде Румынии.

Объяснение неизбежно

Во вторник около 10 часов утра в комнату инспектора Варнике ворвался незнакомец. Он был крайне возбужден. Руки его дрожали, взъерошенные волосы торчали во все стороны. Через несколько минут, закурив сигарету и успокоившись, посетитель начал свой рассказ: — Сегодня утром я вернулся из отпуска. Всю ночь мне пришлось трястись в поезде. Я не выспался и, придя домой, решил прилечь на диван. От усталости я не сразу заметил, что из комнаты исчез рояль, а журнальный столик и кресло сдвинуты с места. На этом листе бумаги я начертил план расположения мебели в комнате до моего отъезда. — Вот что, уважаемый, — сказал инспектор Варнике, бегло взглянув на рисунок, — Прежде всего мне совершенно ясно, что рояля у Вас вообще не было. А теперь давайте выясним, зачем Вам понадобилась эта ложь. Почему инспектор Варнике усомнился в правдивости рассказа посетителя?

ЗАДАЧИ НА ЛОГИКУ

Логические задачи , так же как и математику, называют «гимнастикой ума». Но, в отличие от математики, задачи на логику — это занимательная гимнастика, которая в увлекательной форме позволяет испытывать и тренировать мыслительные процессы, иногда в неожиданном ракурсе. Для их решения нужна сообразительность, иногда интуиция, но не специальные знания. Решение задач на логику состоит в том, чтобы досконально разобрать условие задачи, распутать клубок противоречивых связей между персонажами или объектами. Логические задачи для детей — это, как правило, целые истории с популярными действующими лицами, в которые нужно просто вжиться, почувствовать ситуацию, наглядно ее представить и уловить связи.

Даже самые сложные задачи на логику не содержат чисел, векторов, функций. Но математический способ мышления здесь необходим: главное, осмыслить и понять условие логической задачи . Не всегда самое очевидное решение, лежащее на поверхности, является правильным. Но чаще всего, решение задачи на логику оказывается гораздо проще, чем кажется на первый взгляд, несмотря на путаное условие.

Интересные задачи на логику для детей по самым разным предметам — математике, физике, биологии — вызывают у них повышенный интерес к этим учебным дисциплинам и помогают в их осмысленном изучении. Логические задачи на взвешивание, переливание, задачи на нестандартное логическое мышление помогут и в повседневной жизни решать житейские проблемы нестандартным образом.

В процессе решения задач на логику вы познакомитесь с математической логикой — отдельной наукой, именуемой по-другому «математикой без формул». Логика как наука была создана Аристотелем, который был не математиком, а философом. И логика первоначально была частью философии, одним из методов рассуждений. В труде «Аналитики» Аристотель создал 20 схем рассуждений, которые назвал силлогизмами. Одним из самых известных его силлогизмов является: «Сократ — человек; все люди смертны; значит Сократ смертен». Логика (с др.-греч. Λογική — речь, рассуждение, мысль) — это наука о правильном мышлении, или, иными словами, «искусство рассуждения».

Существуют определенные приемы решения логических задач :

способ рассуждений , с помощью которого решаются самые простые логические задачи. Этот метод считается самым тривиальным. В ходе решения используются рассуждения, последовательно учитывающие все условия задачи, которые постепенно приводят к выводу и правильному ответу.

способ таблиц, применяемый при решении текстовых логических задач. Как следует из названия, решение логических задач заключается в построении таблиц, которые позволяют наглядно представить условие задачи, контролировать процесс рассуждений и помогают сделать правильные логические выводы.

способ графов состоит в переборе возможных вариантов развития событий и окончательном выборе единственно верного решения.

способ блок-схем — метод, широко используемый в программировании и решении логических задач на переливание. Он заключается в том, что сначала в виде блоков выделяются операции (команды), затем устанавливается последовательность выполнения этих команд. Это и есть блок-схема, которая по сути является программой, выполнение которой приводит к решению поставленной задачи.

способ бильярда следует из теории траекторий (один из разделов теории вероятности). Для решения задачи необходимо нарисовать бильярдный стол и интерпретировать действия движениями бильярдного шара по разным траекториям. При этом необходимо вести записи возможных результатов в отдельной таблице.

Каждый из этих методов применим к решению логических задач из разных областей. Эти, казалось бы, сложные и научные приемы вполне можно использовать в решении задач на логику для 1, 2, 3, 4, 5, 6, 7, 8, 9 классов.

Представляем вам самые разнообразные логические задачи для 1, 2, 3, 4, 5, 6, 7, 8, 9 класса. Мы подобрали для вас наиболее интересные задачи на логику с ответами , которые будут интересны не только детям, но и родителям.

  • подбирайте для ребенка задачи на логику в соответствии с его возрастом и развитием
  • не торопитесь открыть ответ, позвольте ребенку самому найти решение логической задачи . Пусть он сам дойдет до правильного решения и вы увидите — какое удовольствие и чувство восторга у него возникнет при совпадении его ответа с данным.
  • в процессе решения задач на логику допустимы наводящие вопросы и косвенные подсказки, указывающие направление размышления.

С помощью нашей подборки логических задач с ответами вы действительно научитесь решать логические задачи, расширите свой кругозор и значительно разовьете логическое мышление. Дерзайте!!!

Решение логических задач — первый шаг к развитию ребенка.

Э.Давыдова

Логика — это искусство приходить к непредсказуемому выводу.

Сэмюэл Джонсон

Без логики почти невозможно внесение в наш мир гениальных находок интуиции.

Кирилл Фандеев

Человек, рассуждающий логично, приятно выделяется на фоне реального мира.

Американское изречение

Логика — это нравственность мысли и речи.

Ян Лукасевич

Как вы относитесь к тому, чтобы на несколько минут отвлечься от серой рутины и немного размять свой мозг? Тогда выбирайте любой интересный вопрос на логику из этой статьи и попробуйте самостоятельно найти на него ответ. Только чур не подглядывать в ответы сразу же — это не только нечестно, но и неинтересно!

Мысленная разминка для малышей

Многие из этих загадок хорошо известны ещё с советских времён, но до сих пор не теряют своей актуальности. Ответы на них настолько просты и очевидны, что сразу догадаться практически невозможно. Приготовились? Тогда полный вперёд!

1. «По чему, когда хочешь спать, идёшь к кровати?» Вся «фишка» этого вопроса заключается именно в формулировке. Ведь если произнести его вслух, мозг сразу воспринимает первые два слова как одно целое. Почему? Ну как это «почему»? На кровать можно лечь, накрыться одеялом, закрыть глаза и… И, кстати, правильный ответ — «По полу».

2. «Когда человек может быть в комнате без головы?» Ещё один вопрос на логику с элементарным ответом. Однако дойти до правильного решения ребёнку может быть весьма непросто, ведь даже не каждый взрослый может сразу догадаться, что такое случается, когда мы высовываем голову в окно.

3. «Может ли страус назвать себя птицей?» Может, вы удивитесь, но никаких особых познаний из области зоологии для того, чтобы правильно ответить на этот вопрос, не нужно, ведь даже самый образованный и эрудированный страус не сможет назвать себя никак. Хотя бы потому, что говорить он не умеет.

4. «В каких словах по сто согласных?» А вот тут ребёнок, несомненно, призадумается. Ведь сложно даже представить себе такое слово — целых 100 согласных, а если ещё и гласные добавить? Что же это за лексический монстр такой? Но правильный ответ, как всегда, лежит на поверхности — «стоЛ», «стоН», «стоЙ», «стоГ», «стоП».

5. «Перед вами — ванна, заполненная водой. На краю лежит кружка и ложка. Что нужно использовать, чтобы побыстрее удалить из ванны всю воду?» Думаете, это кружка? Потому, что она больше? А вот рациональный человек, глядя на ваши мучения, молча подойдёт и вытащит пробку.

6. «По лесу шли три поросёнка. Один шёл впереди двоих, один — позади всех, и один — между двумя. Как они шли?» Честно говоря, на такие вопросы на логику с подвохом нередко не могут ответить даже взрослые люди. На самом же деле, поросята в этой загадке просто идут друг за другом.

7. «Бык весь день пахал поле. Сколько следов он в итоге оставил на пашне?» На самом деле, бык не оставляет следов вообще, ведь плуг, который он тянет за собой, стирает их.

8. «В 12 часов ночи идёт проливной ливень. Может ли быть так, что спустя 72 часа будет тёплая, солнечная погода?» Никакая теория вероятности вам тут не поможет, расслабьтесь. Зато поможет знание того, сколько часов в сутках — солнечной погоды быть не может. Хотя бы потому, что через указанные 72 часа снова будет полночь.

Итак, мы рассмотрели некоторые любопытные вопросы на логику для детей. А сейчас перейдём к другим, более сложным и интересным заданиям.

Другие логические задачки

Предлагаем вашему вниманию другие интересные вопросы на логику, которые могут заставить задуматься не только малышей, но и их родителей.

Игра слов

  • «На морском берегу лежал камень, на поверхности которого было выцарапано слово из 8 букв. Когда это слово читали богатые, они начинали рыдать, бедные — напротив, радовались, а влюблённые пары — расставались. Что это было за слово?». Мы не будем никак комментировать ответ, ведь всё станет ясно само собой. А слово это было — «Временно».
  • «В каком слове есть сразу 3 буквы «л» и 3 буквы «п»? — «Параллелепипед».

Для знатоков математики

  • «Сколько земли в яме диаметром в 3 метра и глубиной в 5 метров?» Всё ещё пытаетесь подсчитать и ищете плотность разных типов почв? Не забывайте, что это — вопрос на логику. Уже по факту своего существования яма пустая, иначе она бы не была ямой.
  • «Сколько раз можно отнять 6 от 30?» Да не поделить же, а отнять! Всего один, ведь в следующий раз вы будете отнимать 6 уже не от 30, а от 24.

Жизненные

  • «Два друга гуляли по городу и вдруг остановились, начав спорить. Один стал утверждать, что «это — красная». Другой возразил ему и сказал, что «это — чёрная». Первый не растерялся и спросил: «Почему же в таком случае она белая?», на что услышал: «Да оттого, что зелёная». О чём они говорили?» Верный ответ на данную загадку — смородина.
  • «Три столетия тому назад эту процедуру проводили на расстоянии 50 метров. Сейчас это расстояние сократилось в 10 раз, а всё благодаря изобретению советского учёного, которое вам наверняка неоднократно приходилось видеть. Что это?» Ничего не приходит в голову? На самом деле, речь идёт о таблице для проверки зрения, также известной как

Американские учёные, которым на глаза попалась эта картинка и вопросы к ней, посчитали, что это — один из самых эффективных тестов на IQ за всю историю человечества. Внимательно рассмотрите изображение, а потом попробуйте дать правильный ответ всего лишь на 9 вопросов.

Вопросы

  1. Сколько туристов остановилось в этом лагере?
  2. Как давно они сюда приехали: сегодня или несколько дней тому назад?
  3. Далеко ли расположен лагерь от ближайшего населённого пункта?
  4. На чём туристы добрались сюда?
  5. Какое сейчас время суток?
  6. Откуда дует ветер: с юга или с севера?
  7. Куда ушёл Шура?
  8. Назовите имя того, кто вчера был дежурным.
  9. Какое сейчас число и какого месяца?

Правильные ответы

Ломаете голову? Ну что ж, время раскрыть карты и продемонстрировать, насколько элементарны ответы даже на самые сложные вопросы на логику:

  1. Четверо. Чтобы понять это, достаточно посмотреть на список дежурных (в нём четыре строки), а также на количество тарелок и ложек на подстилке.
  2. Не сегодня, ведь между деревом и палаткой проворный паучок успел сплести паутинку.
  3. Маловероятно, ведь ребята смогли привезти с собой живую курицу (или она забежала к ним случайно, что, впрочем, не меняет сути).
  4. На лодке. Возле дерева можно увидеть пару вёсел, а так как машин в советское время было не особо много, это — самый логичный вариант ответа.
  5. Утро, ведь тень падает в западную сторону, а стало быть, солнце светит с востока.
  6. Этот вопрос на логику действительно требует дополнительных знаний. К примеру, нужно помнить, что на южной стороне дерева ветки всегда длиннее, чем на северной. Дальше нужно посмотреть на огонь — он немного уклоняется в сторону севера, а значит, ветер дует с юга.
  7. Шура отправился ловить бабочек — из-за кустов можно разглядеть сачок, падающий на крылатую красавицу.
  8. Как видим, Шура ушёл за бабочками, а мальчик, сидящий возле рюкзака с буквой «К» — Коля. То есть, два варианта уже отпадают. Ещё один мальчик занимается тем, что фотографирует окружающую природу. Он также не может быть дежурным. Но как его зовут? Приглядевшись, можно увидеть, что в рюкзаке с буквой «В» лежит штатив — незаменимый атрибут фотографа. Делаем вывод, что имя фотографа начинается с этой же буквы — значит, фотографирует Вася. Методом исключения узнаём, что сегодня дежурит Петя, а отсюда приходим к выводу, что вчера дежурил Коля.
  9. Ответ на этот вопрос тесно связан с предыдущим. Итак, сегодня на дежурстве Петя. Возле его имени на доске написана цифра 8 — 8-е число. Что касается месяца, то сама обстановка на картинке говорит о том, что дело происходит в августе — только тогда в наших широтах появляются арбузы. Конечно, есть они и в сентябре. Но вот бабочек в начале осени найти бывает довольно трудно, а на земле появляется первая опавшая листва.

Интересно? А знаете ли вы, что всего 6% людей могут правильно ответить на все 9 вопросов? Если вам это удалось, можете принять поздравления, ведь это означает, что ваш IQ составляет от 130 и больше.

Логические задачи — пожалуй, самый эффективный инструмент для развития логики и мышления как у детей, так и у взрослых.

Решение задачи на логику предполагает сложный мыслительный процесс. Это последовательное совершение определённых логических действий, работа с понятиями, использование различных логических конструкций, построение цепочки точных рассуждений с правильными промежуточными и итоговыми умозаключениями.

В отличие от большинства математических и других видов задач, при решении логических задач ключевым является не нахождение количественных характеристик объекта, а определение и анализ отношений между всеми объектами задачи.

Используйте комплексный подход

Среди всего многообразия логических задач часто дети выбирают себе пару любимых категорий и погружаются в их решение. Достаточно ли этого?

Наверняка большинство из нас хотя бы раз проходили тесты на уровень логики. Большинство их составлено из одних силлогизмов или вопросов с подвохом. Мы не предлагаем подобные тесты, потому что точно знаем, что определить уровень развития логического мышления с помощью десятка или двух вопросов, даже приблизительно, невозможно. Так же, как и развить нестандартное мышление, решая только отдельные типы логических задач.

Классические логические, комбинаторные и истинностные задачи, закономерности и математические ребусы, задачи про фигуры в пространстве и развертки, на перестановки и движение, на взвешивание и переливание; решаемые с конца, с помощью таблиц, отрезков, графов или кругов Эйлера – это далеко не все разнообразие логических задач, при решении которых активизируются всевозможные мыслительные операции и развивается творческое, нестандартное мышление.

Логика — это вкусняшка для ума

Именно так написали на доске ученики перед началом одного из занятий нашего кружка по логике. В чём же прелесть логических задач?

  • они будут одинаково интересны и увлечённым математикой детям, и «гуманитариям»;
  • многие из них не требуют знаний школьной программы;
  • их может решать даже дошкольник без навыков чтения (например, судоку, ребусы, головоломки со спичками, «шестерёнки» и другие задачи в картинках).

Дети любят решать логические задачи и загадки. Им это интересно! Когда я работала в школе, я видела, что ребята справляются с программой, механически запоминая способ решения тех или иных типовых задач.

А задачи со звёздочками сразу оживляли класс, в процесс обсуждения включались и сильные, и слабые ученики. Дома эту задачу дети уже могли и хотели сами объяснить родителям. Но даже эти задачи со звёздочками были расположены на страницах учебника случайным образом, не было выработано никакой системы.

Битно Галина Михайловна

завуч LogicLike, учитель высшей категории

Только системный и комплексный подход создаёт благоприятные предпосылки для формирования нестандартного мышления. «Пища для ума» тоже должна быть сбалансированной и разнообразной. Попробуйте сами и предложите вашим детям решить именно такую подборку задач. Это поможет выявить те звенья в логике, над которыми стоит поработать усерднее.

Попробуйте сами

В онлайн-платформе Logiclike , созданной для развития логики и математических способностей у детей 5-12 лет, авторы постарались реализовать всё то, чего зачастую так не хватает и ученикам, и учителям в школьных программах. Системность, вовлечение, интерактивность, наглядность, мотивация… Но первым делом это — пища для ума, та самая «вкусняшка», которая заставляет ребенка думать, рассуждать, проверять свои силы, проявлять творческий подход и радоваться, когда удаётся найти правильное решение.

  • Хотите развить у ребенка нестандартное мышление и гибкую логику – давайте ему хорошую зарядку для ума в виде разнообразных логических задач, для решения которых нужно использовать разные логические законы и методы решения (метод с конца, табличный метод, с помощью графов или кругов Эйлера и т.д.)
  • Подходите к обучению системно: от теории к задачам, от простого к сложному, от знакомства с новыми типами заданий к рефлексии.
  • Учитывайте специфику мышления у детей младшего школьного возраста – используйте визуальные образы и наглядные материалы.
  • Важно не навязывать детям способ решения, а стараться проводить разбор так, чтобы они сами путем логических рассуждений нашли правильный ответ.
  • Внедряйте игровые элементы в процесс обучения, используйте обучающие возможности IT.
  • Занятия логикой, как и спортивные тренировки, нуждаются в регулярности и постепенном повышении сложности задач.

Занимайтесь вместе с ребенком и с удовольствием!

20 сложных загадок для взрослых

Кто сказал, что умопомрачительные логические головоломки предназначены только для детей? Мы придумали 20 совершенно новых загадок для взрослых, чтобы проверить ваше критическое мышление, математику и логику. Здесь каждый найдет что-то для себя со сложностями, от легких до умеренных и тяжелых.

🤯 Вы любите сложные загадки. И мы тоже. Давайте вместе справимся с трудностями.

Так что возьмите карандаш и лист бумаги для заметок и приготовьтесь вырвать волосы (и мы действительно имеем в виду это наилучшим образом).Если вы думаете, что получили правильный ответ, щелкните ссылку внизу каждой загадки, чтобы найти решение. Ошибся? Не беспокойтесь, у вас есть еще 19 загадок, которые нужно проверить.


Загадка № 1: Приказы короля превращаются в адскую головоломку

(Сложность: легкая)

Король Нупе из королевства Катан так обожает своих двух дочерей, что решает Королевству было бы лучше, если бы девочек было больше, чем мальчиков, и он издает следующий указ: Все детородные пары должны продолжать рожать детей, пока у них не родится дочь!

Но, чтобы избежать перенаселения, он издает дополнительный указ: все детородные пары откажутся от заводить детей, как только у них родится дочь! Его подданные немедленно начинают выполнять его приказы.

Каково ожидаемое соотношение девочек и мальчиков в Катане по прошествии многих лет?

Подсказка

Вероятность того, что каждый ребенок родится девочкой, конечно, составляет 50 процентов.

Решение

Готовы к решению? Нажмите здесь, чтобы убедиться, что вы правы.



Загадка № 2: Сколько яиц несла эта курица?

(Сложность: Легкая)

Эта задача в честь моего отца, Гарольда Фейвсона.Благодаря ему я люблю решать математические головоломки, и это одна из первых задач (из многих), которые он дал мне, когда я рос.

Курица за полтора дня откладывает полтора яйца. Сколько яиц откладывает одна курица за день?

Решение

Готовы к решению? Нажмите здесь, чтобы убедиться, что вы правы .


Загадка № 3: математическая задача с золотой цепью обманчиво проста

(Сложность: средняя)

Вы роетесь на чердаке своей прабабушки и находите пять коротких цепочек, каждая из которых состоит из четырех золотых звеньев.Вам приходит в голову, что если вы объедините их все в одну большую петлю из 20 звеньев, у вас получится невероятное ожерелье. Итак, вы приносите его ювелиру, который говорит вам, что стоимость изготовления ожерелья будет составлять 10 долларов за каждую золотую связь, которую она должна разорвать, а затем снова запечатать.

Сколько это будет стоить?

Решение

Готовы к решению? Нажмите здесь, чтобы убедиться, что вы правы.


Загадка № 4: Попробуйте решить эту головоломку с пиклболом

(Сложность: «ТРУДНО»)

Кенни, Эбби и Нед собрались на круговой турнир по пиклболу, где, как обычно, победитель остается после каждой игры, чтобы сыграть с тем, кто выбыл из этой игры.К концу полудня, посвященного пиклболу, Эбби устала, сыграв последние семь игр подряд. Кенни, который менее запутан, подсчитывает количество сыгранных игр:

Кенни сыграл восемь игр

Эбби сыграл 12 игр

Нед сыграл 14 игр

Кто кого выиграл в четвертой игре?

Подсказка

Сколько всего игр было сыграно?

Решение

Готовы к решению? Нажмите здесь, чтобы убедиться, что вы правы .


Загадка № 5: Наша загадка с автоматическим выключателем — чистое зло. Извините.

(Сложность: 🚨HARD 🚨 )

Коробка выключателя в вашем новом доме находится в неудобном углу вашего подвала. К своему огорчению, вы обнаруживаете, что ни один из 100 автоматических выключателей не имеет маркировки, и сталкиваетесь с устрашающей перспективой сопоставления каждого автоматического выключателя с соответствующим светом. (Предположим, что каждый автоматический выключатель соответствует только одной лампе.)

Для начала вы включаете все 100 источников света в доме, а затем спускаетесь в подвал, чтобы начать обременительный процесс картирования. При каждой поездке в подвал вы можете включать и выключать любое количество автоматических выключателей. Затем вы можете бродить по коридорам своего дома, чтобы узнать, какой свет включен, а какой выключен.

Какое минимальное количество поездок вам нужно совершить в подвал, чтобы сопоставить каждый автоматический выключатель с каждым светом?

Подсказка

Решение , а не включает в себя включение или выключение выключателей света в вашем доме или определение температуры лампочек.Возможно, вы захотите сначала попробовать решение для случая с 10 автоматическими выключателями без маркировки.

Решение

Готовы к решению? Щелкните здесь, чтобы убедиться, что вы правы .


Загадка №6: Два поезда. Две бабушки. Сможете ли вы решить эту сложную математическую загадку?

(Сложность: средняя)

Две бабушки Джесси хотят видеть его каждые выходные, но они живут на противоположных концах города. В качестве компромисса он говорит им, что каждое воскресенье он будет идти к ближайшей к его квартире станции метро в произвольное время дня и сесть на следующий прибывший поезд.

Если это поезд, идущий на север, он посетит свою бабушку Эрику в верхней части города, а если это окажется поезд, едущий на юг, он посетит свою бабушку Кара в центре города. Обе его бабушки согласны с этим планом, так как они знают, что поезда и на север, и на юг ходят каждые 20 минут.

Но через несколько месяцев бабушка Кара жалуется, что видит его только одно воскресенье из пяти. Джесси обещает, что он действительно направляется на станцию ​​каждый день в случайное время. Как такое может быть?

Подсказка

Поезда всегда прибывают в назначенное время.

Решение

Готовы к решению? Нажмите здесь, чтобы убедиться, что вы правы .


Загадка № 7: Вот чертовски сложная математическая задача о муравьях

(Сложность: «ТРУДНО»)

Макс и Роуз — братья и сестры муравьев. Они любят соревноваться друг с другом, но всегда равны, поскольку на самом деле ползут с одинаковой скоростью.Поэтому они решают устроить гонку, в которой один из них (надеюсь) победит.

В этой гонке каждый из них стартует в нижнем углу кубоида, а затем будет ползти со всей возможной скоростью, чтобы добраться до крошки в противоположном углу. Размеры их кубов, как показано на рисунке:

.

Лаура Фейвесон

Если они оба выберут кратчайший путь, чтобы добраться до своей крошки, , кто первым доберется до своей крошки? (Не забывайте, что это муравьи, поэтому, конечно, они могут лазить по краям или поверхности кубоида.)

Подсказка

Помните: мыслите нестандартно.

Решение

Готовы к решению? Нажмите здесь, чтобы убедиться, что вы правы .


Загадка № 8: Загадка с мятой и пирожком практически невозможна

(Сложность: «СЛОЖНО»)

Вы встречаетесь со своей подругой Кэрин в «конфетке», которая работает следующим образом : Есть стопка из 100 карамелей и одна котлета из мяты. Вы с Кэрин будете ходить взад и вперед, беря по крайней мере одну и не больше пяти карамель из стопки конфет за каждый ход.Тот, кто уберет последнюю карамель, также получит пирожок с перечной мятой. А вы любите пирожков с мятой.

Предположим, Кэрин позволяет вам решать, кто пойдет первым. Кого выбрать, чтобы выиграть пирожок с перечной мятой?

Подсказка

Сначала решите кучу из 10 карамель.

Решение

Готовы к решению? Нажмите здесь, чтобы убедиться, что вы правы .


Загадка № 9: Сможете ли вы разгадать великую загадку американской железной дороги?

(Сложность: Средняя)

Эту задачу предложил физик П.Джеффри Ангар.

Наконец, Great American Rail-Trail по всей стране завершен! Идите, похлопайте себя по спине — вы только что установили самый длинный поручень в истории мира, протяженностью 4000 миль от начала до конца. Но сразу после церемонии открытия ваш помощник напоминает вам, что металл, который вы использовали для поручня, летом немного расширяется, так что его длина в общей сложности увеличивается на один дюйм.

«Ха!» вы говорите: «Один дюйм в перилах на 4000 миль? Ничего подобного! Но… ты прав?

Предположим, что когда поручень расширяется, он выгибается вверх в самом слабом месте, которое находится в центре.Насколько выше пешеходы в центре страны должны будут подниматься летом, чтобы ухватиться за поручни? То есть на рисунке ниже что такое h ? (Для целей этого вопроса игнорируйте кривизну Земли и предполагайте, что след представляет собой прямую линию.)

Лаура Фейвесон

Подсказка

Пифагор — увлекательная историческая личность.

Решение

Готовы к решению? Нажмите здесь, чтобы убедиться, что вы правы.


Загадка № 10: эта загадка похожа на особо жестокую задачу SAT. Сможете ли вы найти ответ?

(Сложность: средняя)

Аманда живет со своим сыном-подростком Мэттом в сельской местности — на машине от школы Мэтта. Каждый день Аманда в одно и то же время покидает дом, едет в школу с постоянной скоростью, забирает Мэтта именно тогда, когда его шахматный клуб заканчивается в 17:00, а затем они немедленно возвращаются домой с той же постоянной скоростью.Но однажды Мэтт плохо себя чувствует, поэтому он рано бросает шахматную тренировку и отправляется домой на своем портативном самокате.

После того, как Мэтт бежал в течение часа, Аманда встречает его в своей машине (на своем обычном пути, чтобы забрать его), и они возвращаются вместе, прибыв домой на 40 минут раньше, чем обычно. Сколько шахматных тренировок пропустил Мэтт?

Подсказка

Рассмотрим случай, когда Аманда встречает Мэтта именно тогда, когда она выходит из их дома.

Решение

Готовы к решению? Нажмите здесь, чтобы убедиться, что вы правы .


Загадка № 11: Сможете ли вы переправить этих трех кинозвезд через реку?

(Сложность: средняя)

Три кинозвезды, Хлоя, Лекса и Джон снимают фильм в Амазонке. Они очень известны и требуют особого ухода, поэтому их агенты всегда с ними. Однажды, после съемок сцены в глубине тропического леса, трое актеров и их агенты решают вернуться на базу пешком. Вдруг они подходят к большой реке.

На берегу реки они находят небольшую весельную лодку, но она достаточно велика, чтобы вместить две лодки одновременно. Уловка? Никому из агентов не нравится оставлять свою кинозвезду с другими агентами, если их тоже нет. Они не верят, что другие агенты не попытаются переманить их звезду.

Например, агент Хлои в порядке, если Хлоя и Лекса одни в лодке или на одном из берегов реки, но определенно не , если агент Лексы тоже с ними. Так как же им всем перебраться через реку?

Подсказка

Эту проблему нельзя решить только одним способом.

Решение

Готовы к решению? Щелкните здесь, чтобы убедиться, что вы правы .


Загадка № 12: Эта до смешного сложная загадка — наша дань уважения позднему гению математики. Вы можете это понять?

(Сложность: 🚨HARD 🚨)

11 апреля Джон Хортон Конвей , блестящий математик, который страстно и игриво любил головоломки и игры, умер от осложнений после COVID-19.Конвей — изобретатель одной из моих любимых легендарных задач (не для слабонервных) и, как известно, Game of Life . Я создал эту задачу в его честь.

Кэрол создавала генеалогическое древо, но у нее возникли проблемы с отслеживанием даты рождения своей матери. Единственным ключом к разгадке, который она нашла, было письмо, написанное ее дедушкой бабушке в день рождения ее матери. К сожалению, некоторые символы были размыты, представленные здесь «___» .(Длина линии не отражает количество нечетких символов.)

«Дорогая Вирджиния,

» Мало ли, когда я направлялся на работу в понедельник утром, что к вечеру у нас родится красивая девочка. И в годовщину нашей свадьбы, не меньше! Это заставляет меня вспомнить тот невероятный выходной день, J___ 27, 19___ , когда мы впервые поделились нашей клятвой создать вместе семью, и, ну, вот и мы! С восьмилетним юбилеем, любовь моя.

С любовью, Эдвин »

Вопрос: когда родилась мать Кэрол?

Подсказка

Эта задача основана на правиле Судного дня Конвея.

Решение

Готовы к решению? Щелкните здесь, чтобы убедиться, что вы правы .


Загадка № 13: Чтобы разгадать эту запутанную математическую загадку, вам нужен только один пояс и одна земля

(Сложность: средняя)

Представьте, что у вас очень длинный пояс. Что ж, очень долго, правда … на самом деле, этого достаточно, чтобы он мог плотно обернуть вокруг всей нашей планеты. (Для простоты предположим, что Земля идеально круглая, без гор, океанов или других препятствий на пути пояса.)

Естественно, вы очень гордитесь своим поясом. Но тут появляется ваш брат Питер — и, к вашему недовольству, он производит ремень , всего на немного длиннее вашего. Он хвастается, что его пояс длиннее ровно на его рост: 6 футов.

Если бы Питер также обернул свой пояс по окружности Земли, как далеко над поверхностью он смог бы подвесить пояс, если бы натянул его туго и равномерно?

Подсказка

Окружность Земли составляет около 25 000 миль или 130 миллионов футов … но вам не нужно знать это, чтобы решить эту проблему.

Решение

Готовы к решению? Щелкните здесь, чтобы убедиться, что вы правы .


Загадка № 14: Эта загадка, касающаяся локтей, дьявольская. Удачи в решении.

(Сложность: 🚨HARD 🚨 )

Когда-нибудь в будущем, когда будут сняты запреты на приюты на месте, супружеская пара Флориан и Джулия направятся в бар, чтобы отпраздновать свое новообразование. Свобода.

Там они находят еще четыре пары, которым пришла в голову такая же идея.

Стремясь к социальному контакту, каждый человек в пяти парах с энтузиазмом постукивает локтями (новое рукопожатие) каждому человеку, которого он еще не встретил .

На самом деле оказывается, что многие люди знали друг друга раньше, поэтому, когда Джулия спрашивает всех, сколько локтей они постучали, она получает девять разных ответов!

Вопрос: сколько локтей Флориан постучал?

Подсказка

Какие девять ответов услышала Джулия?

Решение

Готовы к решению? Щелкните здесь, чтобы убедиться, что вы правы .


Загадка № 15: Вам понадобится выпить после попытки разгадать эту загадку виски

(Сложность: Легко)

Алан и Клэр живут согласно старой шотландской поговорке: « Ни виски без воды, ни воды без виски! » Итак, однажды, когда перед Аланом стоит стакан виски, а перед Клэр стоит стакан воды такого же размера, Алан берет ложку своего виски и наливает ее в воду Клэр.

Клэр перемешивает воду с оттенком виски, а затем кладет ложку этой смеси обратно в виски Алана, чтобы убедиться, что они выпили ровно столько же.

Итак: есть ли больше воды в виски Алана или больше виски в воде Клэр? И имеет значение, насколько хорошо Клэр помешалась?

Подсказка

Размер ложки не имеет значения .

Решение

Готовы к решению? Щелкните здесь, чтобы узнать, правы ли вы.


Загадка №16: Проблема дудла намного сложнее, чем кажется. Сможете ли вы это решить?

(Сложность: средняя)

Загадка этой недели относительно проста, но все равно зловеща.

Вопрос: можно ли получить 100, вставляя любое количество плюсов и минусов в строку цифр 9 8 7 6 5 4 3 2 1? Вы не можете изменить порядок цифр ! Итак, какое количество плюсов и минусов необходимо из , чтобы получить 100?

Эндрю Дэниелс


Например, 98-7-6 + 54-32 показывает один способ перемежать плюсы и минусы, но, поскольку он равен 107, это не решение.

Я называю это «проблемой каракули»: над ней лучше всего работать во время встреч, где в противном случае вы могли бы рисовать.

Подсказка

Возможно, вы захотите начать поиск решений, которые используют в общей сложности семи плюсов и минусов (хотя есть способы использовать меньше).

Решение

Готовы к решению? Щелкните здесь, чтобы узнать, правы ли вы.


Загадка № 17: Эта математическая головоломка озадачила всех ученых, кроме одного.Думаешь, ты сможешь это взломать?

(Сложность: HARD)

В честь Фримена Дайсона, известного физика, который умер в прошлом месяце , вот легендарная история, демонстрирующая его сообразительность и невероятную силу ума.

Однажды на собрании ведущих ученых один из них вслух поинтересовался, существует ли целое число, которое можно было бы точно удвоить, переместив последнюю цифру на передний план. Например, 265 удовлетворял бы этому , если бы 526 было его точным двойником, а это не так.

После , очевидно, всего за пять секунд , Дайсон ответил: «Конечно, есть, но самое маленькое такое число состоит из 18 цифр».

Это оставило некоторых самых умных ученых в мире недоумением, как он мог это выяснить так быстро.

Итак, учитывая подсказку Дайсона, какое наименьшее такое число?

Подсказка

Мой второклассник недавно научился складывать себе трехзначное число с помощью классического вертикального метода:

Эндрю Дэниелс

18-значных чисел, конечно, можно сложить таким же образом.

Решение

Готовы к решению? Нажмите здесь, чтобы убедиться, что вы правы .


Загадка № 18: выяснить, что у нее на лбу

(Сложность: средняя)

Сесилия любит проверять логику своих очень логичных друзей Джайи, Джулиана и Леви, поэтому она объявляет :

«Я напишу положительное число на каждом твоем лбу. Ни одно из чисел не совпадает, и два числа в сумме дают третье.

Она пишет числа на их головах, затем поворачивается к Джайе и спрашивает ее, какой у нее номер. Джая видит, что у Джулиана 20 на лбу, а у Леви 30 на лбу. Она задумывается на мгновение, а затем говорит: «Я не знаю, какой у меня номер». Джулиан вставляет: «Я тоже не знаю своего номера», а затем Леви восклицает: «Я тоже!» Сесилия радостно говорит: «Я наконец-то поставила вас в тупик, ребята!»

«Не так быстро!» — говорит Джая. «Теперь я знаю свой номер!»

Какой номер у Джайи?

Подсказка

Джая может быть одним из двух чисел, но только одно из этих чисел приведет к тому, что Джулиан и Леви не будут знать своих чисел.Почему?

Решение

Готовы к решению? Нажмите здесь, чтобы убедиться, что вы правы .


Загадка № 19: Сможете ли вы добиться избрания Киану Ривза президентом?

(Сложность: средняя)

На дворе 2024 год, на демократических праймериз баллотируются пять кандидатов: Тейлор Свифт, Опра Уинфри, Марк Кьюбан, Киану Ривз и Дуэйн Джонсон. (Эй, это может случиться.) Как обычно, первые первичные выборы проходят в Айове.

Стремясь преодолеть замешательство после фиаско фракционного собрания 2020 года, Демократическая партия Айовы только что объявила о новом надежном способе поиска лучшего кандидата: выборы будут проводиться четыре раза подряд.

Сначала кандидат 1 будет баллотироваться против кандидата 2. Затем победитель этого конкурса будет баллотироваться против кандидата 3, затем этот победитель будет баллотироваться против кандидата 4, и, наконец, победитель этих выборов будет баллотироваться против последнего кандидата. По переходному свойству победитель последних выборов должен быть лучшим кандидатом… так говорит Демократическая партия Айовы.

Кандидат Киану чувствует себя довольно плохо, поскольку он знает, что большинство избирателей оценивают его ближе к низу, а к верхнему — нет. Фактически, он знает, что население Айовы разделено на пять равных групп и их предпочтения следующие:

.

Киану дружит в детстве с Биллом С. Престоном, эсквайром, новым главой Демократической партии Айовы. Престон, уверенный, что порядок кандидатов не имеет значения для результата, говорит Киану, что он может выбирать порядок голосования кандидатов.

Итак, какой порядок выбрать Киану?

Подсказка

Как Киану будет выступать в индивидуальных гонках против каждого кандидата?

Решение

Готовы к решению? Нажмите здесь, чтобы убедиться, что вы правы .


Загадка № 20: Кто открыл все эти проклятые шкафчики?

(Сложность: средняя)

В главном коридоре средней школы Хелма есть 100 шкафчиков.Каждую ночь директор школы следит за тем, чтобы все шкафчики были закрыты, чтобы следующий день был организован. Однажды 100 озорных студентов решают, что они разыграют.

Все ученики встречаются перед началом школы и выстраиваются в очередь. Затем первый ученик идет по коридору и открывает все шкафчики. Следующий ученик закрывает все остальные шкафчики (начиная со второго). Затем ученик 3 подходит к каждому третьему шкафчику (начиная с третьего) и открывает его, если он закрыт, и закрывает его, если он открыт.Учащийся 4 открывает каждый четвертый шкафчик, если он закрыт, и закрывает его, если он открыт. Это продолжается и продолжается до тех пор, пока Студент 100, наконец, не дойдет до сотого шкафчика. Когда директор приходит позже утром, какие шкафчики она находит открытыми?

Совет

Обязательно обратите внимание на все факторы.

Решение

Готовы к решению? Нажмите здесь, чтобы убедиться, что вы правы .


🎥 Смотри:


Лаура Фейвесон Лаура Фейвесон — правительственный экономист, рассказчик и на протяжении всей жизни увлекается математическими головоломками.Она живет в Вашингтоне, округ Колумбия, с мужем и двумя дочерьми.

Этот контент создается и поддерживается третьей стороной и импортируется на эту страницу, чтобы помочь пользователям указать свои адреса электронной почты. Вы можете найти больше информации об этом и подобном контенте на сайте piano.io.

Логическая головоломка, которую можно решить только со своим лучшим другом — такие романтичные факты

Чтобы разгадать эту смертельную загадку, нужно умело строить «общие знания». Wikicommons

Вас поймали на жутком кладбище со своим лучшим другом. Смотритель, скучающий старик, любит загадки (и не очень любит нарушителей), заключает каждого из вас в отдельную комнату внутри сарая и, забирая ваши телефоны, говорит: «Только ваш разум может освободить вас». Он показывает вам на зарешеченное окно. Через него можно увидеть 12 статуй. Из окна вашей подруги, выходящего на противоположную сторону кладбища, она видит восемь.Никто из вас не знает счета другого.

Смотритель говорит каждому по отдельности, что вместе вы можете увидеть 18 или 20 статуй. К сожалению, невозможно сказать другу, сколько вы видите. Единственный способ сбежать для вас обоих — это дать одному из вас общее количество видимых статуй. Сделай это неправильно, и никто из вас никогда не уйдет. Смотритель спрашивает вас каждого по отдельности, раз в день, и вы можете выбрать, ответить или пройти. Вы оба знаете, что вас всегда спрашивают первым.* Если вы оба умираете в определенный день, вопрос — 18 или 20? — снова задается каждому из вас на следующий день, а затем и так далее, пока вы не поймете это правильно или неправильно. Смотритель хихикает: «Если я тебе понадоблюсь, я приготовлю твои могилы».

Как вам сбежать?

Если вы хотите разгадывать это самостоятельно, не читайте дальше. Решение следует.

Фотография Юсси-Теппо Тойвонен / Flickr

Преш Талвалкар, автор книги The Joy of Game Theory: An Introduction to Strategic Thinking , подробно изложил решение проблемы в прошлом году в своем блоге «Mind Your Decisions.Загадка была опубликована без указания авторства на форуме, организованном инженером Стэнфордского университета и соучредителем Quantitative Engineering Design Уильямом Ву, пользователем, который слышал о загадке только на шведском языке. Первоначально он изображал злого короля и деревья во дворе замка, но здесь он представлен в более современном виде.

Чтобы пережить столкновение с сумасшедшим смотрителем, ему, возможно, придется накормить вас, если только вы оба не были схвачены с очень полным желудком. Это потому, что в худшем случае потребуется пять дней заключения, чтобы сбежать из тюрьмы.Почему?

Во-первых, вы и ваш друг должны понять, что каждый «проход» считается сигналом . Учитывая, что у вас не было времени посоветоваться друг с другом перед разлукой, это потребует некоторой степени интеллекта — и уверенности друг в друге. Во-вторых, потребуется пять дней подачи сигналов, чтобы точно определить количество статуй. С точки зрения теории игр, это устанавливает необходимое «общее знание» для побега. Многие логические загадки основаны на этом понятии, например, «Голубые глаза», описанном на сайте xkcd.com — как определить цвет своих глаз, если другие люди не говорят вам об этом, и вы не можете видеть свое отражение? Социальный ученый Саймон ДеДео описывает еще одну логическую головоломку, решение которой опирается на построение общих знаний в своей статье Nautilus «Биткойн-парадокс».

День 1

Если вы видели 19 или 20 статуй, то вы можете заключить, что статуй 20. Но вы видите только 12, так что вы проходите мимо. Это сигнализирует вашему другу, что вы видите не более 18 статуй.

Если ваша подруга увидела 0 или 1 статую, зная, что вы видите не более 18, она могла бы сделать вывод, что статуй должно быть 18.Но твой друг видит 8, значит проходит. Это сигнализирует вам, что она видит как минимум 2 статуи.

День 2

Если вы видели 17 или 18 статуй, то вы можете сделать вывод, что их 20, потому что ваш друг должен увидеть как минимум 2 статуи. Но вы видите только 12, так что вы проходите мимо. Это сигнализирует вашему другу, что вы видите не более 16 статуй.

Теперь, если ваша подруга увидела 2 или 3 статуи, зная, что вы видите не более 16, она могла бы сделать вывод, что статуй должно быть 18. Но ваша подруга видит 8 статуй, поэтому она должна пройти.Это сигнализирует вам, что она видит как минимум 4 статуи.

День 3

Если вы видели 15 или 16 статуй, то вы можете сделать вывод, что их 20, потому что ваш друг должен увидеть как минимум четыре статуи. Но вы видите только 12, так что вы проходите мимо. Это сигнализирует вашему другу, что вы видите не более 14 статуй.

Теперь, если ваша подруга увидела четыре или пять статуй, зная, что вы видите не более 14 статуй, она могла бы сделать вывод, что статуй должно быть 18. Но она видит восемь статуй, поэтому ей снова нужно пройти.Это сигнализирует вам, что ваш друг видит как минимум шесть статуй.

День 4

Если вы видели 13 или 14 статуй, то вы можете сделать вывод, что их 20, потому что ваш друг должен увидеть как минимум шесть статуй. Но вы видите только 12, так что вы проходите мимо. Это сигнализирует вашему другу, что вы видите не более 12 статуй.

Теперь, если ваша подруга увидела шесть или семь статуй, зная, что вы видите не более 12 статуй, она могла бы сделать вывод, что статуй должно быть 18. Но ваша подруга видит восемь статуй, поэтому ей снова нужно пройти.Это сигнализирует вам, что она видит как минимум восемь статуй.

День 5

Поскольку ваш друг видит как минимум восемь статуй, а вы видите 12, вы знаете, что их 20. Вы угадаете 20 и получите свободу!

Брайан Галлахер — редактор блога «Факты так романтичны», Nautilus . Следуйте за ним в Twitter @ brianga11agher.

* Этот пост был отредактирован, чтобы уточнить, что вопрос , а не задается одновременно, и что оба заключенных знают, кого спрашивают первым.

ЧАСЫ: Как определить интеллект в терминах кубика Рубика.

веселых логических головоломок

веселых логических головоломок

思 方 網 • сеть критического мышления

(∀ ∴ & модели; ≠ ↔ ⊥ ∃)


Вот несколько коротких, но забавных логических головоломок. Посмотри, сможешь ли ты их решить!

1. Стивен смотрел на фотографию.Кто-то спросил его: «На чью фотографию ты смотришь?» Он ответил: «У меня нет брата или сестры, но отец этого человека — сын моего отца». Итак, на чью фотографию смотрел Стивен?

отвечать

2. Произошло ограбление, при котором было украдено много товаров. Грабитель уехал на грузовике. Известно, что: (1) Никто другой не мог быть замешан, кроме A, B и C. (2) C никогда не совершает преступления без участия A. (3) Б не умеет водить машину. Итак, виноват А. или невиновен?

ответ

3.Предположим, есть этот городок с ограниченным числом людей: (1) Нет двух жителей с одинаковым количеством волос. (2) Ни у одного жителя нет ровно 409 волос. (3) Жителей больше, чем волос на голове у любого жителя. Итак, какое максимальное количество жителей в этом городке?

ответ

4. Следующий тест на логическую дедукцию, как сообщается, был одним из вопросов, который задавали 14-летним школьникам на олимпиаде по математике в Сингапуре и азиатских школах.[Подробнее]

Альберт и Бернард только что подружились с Шерил, и они хотят знать, когда у нее день рождения. Шерил дает им список из 10 возможных свиданий.

  • 15 мая, 16 мая, 19 мая
  • 17 июня, 18 июня
  • 14 июля, 16 июля
  • 14 августа, 15 августа, 17 августа

Затем Шерил сообщает Альберту и Бернарду отдельно месяц и день своего дня рождения соответственно.

Тогда Альберт сказал: я не знаю, когда у Шерил день рождения, но я знаю, что Бернар тоже не знает.

Бернард: Сначала я не знал, когда у Шерил день рождения, но теперь знаю.

Альберт: Тогда я также знаю, когда у Шерил день рождения.

Так когда у Шерил день рождения?

отвечать

5. Эта головоломка принадлежит покойному логику и математику Раймонду М. Смулляну: дилер купил предмет за 7 долларов, продал за 8 долларов, выкупил за 9 долларов и продал за 10 долларов. Какую прибыль он получил? ответ


Пазлы рыцарей и лжецов

Хотите еще головоломок? Попробуйте наши головоломки рыцарей и мошенников, в которых вам нужно выяснить, кто лжет, а кто говорит правду.Их 382 штуки!


Математик утверждает, что большинство людей не могут решить головоломку «3 шляпы»

Этот логический тест о коте в шляпе вызывает недоумение у большинства людей, но запутает ли он вас?

По словам специалиста по экономике и математике Преша Талвалкара с канала YouTube Mind Your Decisions, опрос показал, что только 36 процентов людей могут найти правильный ответ на эту, казалось бы, простую проблему.

Он гласит:

«Есть три шляпы, каждая с сопроводительным заявлением.

Первая шляпа: Кот в этой шляпе.

Вторая шляпа: Кота нет в этой шляпе.

Третья шляпа: Кот не в первой шляпе.

Верно одно из утверждений. Ровно в одной шляпе изображен кот. В какой шляпе изображен кот? »

Варианты ответов: 1) Первая шляпа; 2) Вторая шляпа; 3) Третья шляпа; 4) Ни одной шляпы; или 5) Недостаточно информации.

Хорошо, поэтому возможно, эта проблема не так проста, как кажется.Но, к счастью, Талвалкар сломал, как решить логическую проблему в новом видео на YouTube. Итак, что — это правильный ответ?

Ну, во-первых, вот как решить проблему: вы должны логически рассмотреть каждый случай, предполагая, что кошка в каждой шляпе, а затем посмотреть, применимо ли каждое утверждение к этому случаю. Если вы получите одно истинное утверждение и два ложных утверждения, у вас правильное размещение кошки в шляпе.

Итак, предположим, что кошка в первой шляпе. Заявление Hat One очевидно верно в этом сценарии.Но если кошка в первой шляпе, кошка будет , а не в второй шляпе, поэтому второе утверждение также верно. Это означает, что кота нет в первой шляпе, потому что если бы это было так, два утверждения были бы верными, а это явно не удовлетворяет условиям задачи.

А что, если предположить, что кошка в третьей шляпе? Тогда утверждение Hat Three будет верным, а утверждение Hat One — ложным. Пока что годится только для одного верного утверждения из всей группы. Но проблема возникает при рассмотрении заявления Hat Two: что кота нет в Hat Two.Это было бы также и , если бы кошка была в третьей шляпе. С двумя истинными утверждениями это неправильный ответ.

Спойлер: кот во второй шляпе — и вот почему. Предположим, что кошка в второй шляпе, утверждение, соответствующее этой шляпе, неверно. Кроме того, первое утверждение также неверно, поскольку кошка находится во второй шляпе, а не в первой. Таким образом, истинное утверждение — это утверждение Hat Three. Кота нет в первой шляпе. Этот ответ удовлетворяет условиям запутанности задачи, помещая кота во вторую шляпу с правильным утверждением в третьей шляпе.

Поверьте мне, просмотр проблемы, разыгрываемой в видео Талвалкара, помогает понять этот сложный логический тест. Профессиональный математик говорит, что большинство людей сталкиваются с проблемами, предполагая, что кошка должна быть в шляпе, где утверждение верно. Но это явно не так. Эти два условия следует рассматривать как независимые, чтобы правильно решить проблему.

С учетом всего сказанного, я лично просто собирал каждую шляпу, пока не нашел свою чертову кошку, но, думаю, это не так впечатляет.

Кэти Дюпере Кэти Дюпере — редактор и писатель из Нью-Йорка, специализирующаяся на идентичности, интернет-культуре, социальном благе, образе жизни и красоте.

Этот контент создается и поддерживается третьей стороной и импортируется на эту страницу, чтобы помочь пользователям указать свои адреса электронной почты. Вы можете найти больше информации об этом и подобном контенте на сайте piano.io.

15 сложных загадок с ответами — сложные загадки для проверки вашего разума

Логические загадки, какими бы сложными они ни были, позволят вам подумать о давно забытых вещах, вспомнить сюжет многих детских сказок и принять тот факт, что ваш ребенок получает ответ быстрее, чем вы.Хотите это проверить? Мы подготовили для вас специальный набор сложных логических загадок.

15 сложных логических загадок

Попробуем отгадать каверзные загадки. Дети и взрослые могут искать ответ вместе.

Загадка # 1:

Какие камни нельзя найти в реке?

Показать ответ

Ответ: Сухие камни нельзя найти в реке. Что касается драгоценных камней, иногда они могут.

Загадка № 2:

На столе две монеты, всего три цента.Одна из них не одноцентовая. Какие есть монеты?

Показать ответ

Ответ: На столе лежит монета в два цента и монета в один цент. В заявлении мы считаем, что только одна из монет не одноцентовая.

Загадка № 3:

Где впервые нашли картофель?

Показать ответ

Ответ: Ответ прост — в земле.

Загадка № 4:

Что не поместится даже в самый большой горшок?

Показать ответ

Ответ: Крышка этого горшка.Не ходите на кухню проверять — наверняка застрянет!

Загадка # 5:

Что может быть, стоя и идя, стоя и держась, идя и лежа одновременно?


Вам также может понравиться:


Загадка № 6:

Можно нанести удар, но нельзя нанести удар. Что это?

Показать ответ

Ответ: Это даже не парус. Это разговор.

Загадка № 7:

Я — вода, и я плыву по воде. Кто я?

Загадка № 8:

Ползут три черепахи. Первая черепаха говорит: за мной ползают две черепахи. Вторая черепаха говорит: одна черепаха ползет за мной, а другая передо мной. А третья черепаха говорит: передо мной ползают две черепахи, а за мной еще одна. Как это может быть?

Показать ответ

Ответ: Черепахи кругами ползают!

Загадка № 9:

На ферме были две лошади, один кролик, один щенок, одна кошка, свинья и поросенок, корова и теленок, индейка и гусь.Хозяин приехал с собакой. Сколько футов на ферме?

Показать ответ

Ответ: 24 фута. Почему так? Потому что только у людей, лошадей, свиней и коров есть лапы, а у других животных на ферме лапы есть.

Загадка № 10:

Вы говорите, чтобы он не приходил, но он все равно приходит. Вы говорите, чтобы он не уходил, но он все равно проходит. Что это?

Загадка № 11:

Вы в самолете.Перед вами лошадь, а позади вас машина. Где ты?

Показать ответ

Ответ: Вы едете на карусели.

Загадка № 12:

Иногда идет дождь со странным потоком воды: он бьет вверх сотней струй. Что это?

Загадка # 13:

Чем больше вы берете из него, тем он становится больше. Что это?

Загадка № 14:

Что принадлежит вам, хотя другие используют это чаще, чем вы?

Загадка # 15:

Сколько яиц можно съесть натощак?

Показать ответ

Ответ: Ваш аппетит тут ни при чем.Натощак вы можете съесть только одно яйцо, потому что все следующие натощак нельзя есть.

Зачем нужно разгадывать логические загадки?

При разгадывании логических загадок и поиске ответа задействованы оба полушария мозга. Левое полушарие занимается логикой и пытается понять причинно-следственные связи. Правый имеет дело с интуицией, пытается построить целостную картину и выстраивает творческое мышление.

Большинство современных людей почти никогда не задействуют левое полушарие в повседневной работе; позвольте искать необходимые факты с помощью Google и продолжать свою повседневную работу.Но, конечно, ваши «логические мышцы» можно и нужно тренировать, и здесь нужны логические загадки, поскольку они помогают нам поддерживать мозг в тонусе даже в наши самые ленивые дни.

Зачем детям нужно развивать логику?

Глубокое логическое мышление поможет вашему ребенку легко справиться с математическими задачами как в начальной, так и в средней школе.

Умение анализировать информацию поможет ему или ей на уроках литературы. Умение нестандартно мыслить всегда необходимо на творческих занятиях и школьных мероприятиях.И, конечно же, он всегда сможет доказать свое мнение, отвечая на каверзные вопросы учителя или поспорив с одноклассниками, непременно завоевывая авторитет в школе!

Курсы математики и логики для детей

В

раз сложнее, чем в школе, чрезвычайно весело, интерактивно и полезно, чтобы дети 7-13 лет были вовлечены. Мы заставим их полюбить математику!

учить больше

Логические вопросы и проблемы

Вилка в дороге — вопросы по логике

Вы путешествуете по проселочной дороге в далекую деревню.Вы добираетесь до развилки дороги и находите там пару идентичных сестер-близнецов.

  • Один стоит на дороге в деревню, а другой стоит на дороге в Neverland (конечно, вы не знаете и не видите, куда ведет каждая дорога).
  • Одна из сестер всегда говорит правду, а другая всегда лжет (конечно, вы не знаете, кто лжет).
  • Обе сестры знают, куда ведут дороги.

Если вам разрешено задать только один вопрос одной из сестер, чтобы найти правильную дорогу в деревню, каков ваш вопрос?

Это одна из самых известных логических задач , которая может быть решена с помощью классических логических операций. Возможно, вы слышали несколько вариантов этой головоломки раньше (например, 2 двери — 1 в рай и 1 в ад), но, тем не менее, это одна из лучших головоломок.
Есть несколько типов логических вопросов:
  1. Косвенный вопрос: «Здравствуй, красавица, что бы сказала твоя сестра, если бы я спросила ее, куда ведет эта дорога?» Ответ всегда отрицательный.
  2. Коварный вопрос: «Простите, леди, а на дороге в деревню стоит правдивый человек?» Ответ будет ДА, если я спрашиваю правдивого, стоящего на дороге в деревню, или если я спрашиваю лжеца, снова стоящего на той же дороге.Так что я могу пойти этим путем. Аналогичный вывод можно сделать и для отрицательного ответа.
  3. Сложный вопрос: «Эй, ты, что бы ты сказал, если бы я спросил тебя …?» Говорящий правду ясен, но лжец должен лгать. Однако вопрос заставляет ее дважды солгать и, таким образом, говорить правду.

Честники и мошенники I

На загадочном острове есть два типа людей.Есть так называемые честные люди, которые всегда говорят правду, а другие — мошенники, которые всегда лгут.
Трое парней (A, B и C) ссорятся на рынке. Мимо проходит гринго и спрашивает человека А: «Ты честный или мошенник?» Ответ непонятен, поэтому гринго задает B еще один вполне логичный вопрос: «Что сказал A?» B отвечает: «A сказал, что он мошенник». И на это парень С говорит: «Не верь Б, он лжет!»
Кто такие B и C?

Честники и мошенники II

Потом он встречает еще двух аборигенов.Один говорит: «Я мошенник, другой — Честный».
Кто они?

Честники и мошенники III

Наш гринго рассердил государя своими назойливыми вопросами и был приговорен к смертной казни. Но был и шанс спастись, решив следующую логическую задачу. Гринго показали две двери: одна ведет к эшафоту, а вторая — к свободе (обе двери были одинаковыми), и только охранники знали, что находится за дверями.Государь позволил гринго задать один вопрос одному стражнику. А поскольку государь был честным человеком, он предупредил, что один стражник — мошенник.
Какой логический вопрос может спасти жизнь гринго?
Вы, наверное, помните ответ из самой первой проблемы на этой странице, не так ли 🙂

Честники и мошенники IV

Нашему гринго повезло, и он выжил.По пути в паб он встретил трех аборигенов. Один из них заявил: «Мы все мошенники». Второй заключил: «Один из нас — честный человек».
Кто они?

Честники и мошенники V

В пабе гринго встретил забавного парня, который сказал: «Если моя жена Честная, значит, я Суиндлкант».
Кто эта пара?

Честники и мошенники VI

Когда гринго хотел заплатить и покинуть паб, бармен сказал ему, сколько стоит его напиток.Это было довольно дорого, поэтому он спросил бармена, правда ли он говорит. Но гринго не услышал шепотом ответа, поэтому он спросил об этом человека, сидящего рядом с ним. И человек сказал: «Бармен сказал да, но он большой лжец».
Кто они?

Честники и мошенники VII

Выйдя из паба, гринго услышал о фантастическом закопанном сокровище.Он хотел быть уверенным, поэтому спросил другого человека, который ответил:
«На этом острове — сокровище, только если я честный человек».
Так он пойдет и найдет сокровище?

Честники и мошенники VIII

Думая о сокровище, гринго забыл, какой сегодня день, поэтому он спросил четырех аборигенов и получил следующие ответы:
О: Вчера была среда.
Б: Завтра воскресенье.
C: Сегодня пятница.
Д: Позавчера был четверг.
Потому что все, что вам нужно знать, это сколько людей солгали, я не скажу. Какой это был день недели?

Честники и мошенники IX

После тяжелого дня гринго захотелось немного расслабиться.Но через несколько минут с ним захотели поговорить двое аборигенов. Чтобы прояснить ситуацию, гринго спросил: «По крайней мере, один из вас честный человек?» После ответа не было никаких сомнений.
Кто они и кто ответил?

Честники и мошенники X

На этом острове была девушка, и все хотели ее. Однако ей нужен был только богатый мошенник. Если бы вы были богатым мошенником, как бы вы убедили ее сказать всего одну фразу? А что, если бы она хотела богатого честного человека (и если бы вы были одним из них).Предположим для этой логической задачи, что на острове есть только богатые или бедные люди.

Проблемы логики в суде I

А теперь несколько дел с острова честных и мошенников. Заключенному в баре разрешили произнести одну фразу для защиты. Через некоторое время он сказал: «Преступление совершил мошенник».
Это его спасло?

Проблемы логики в суде II

Человек, обвиняемый в преступлении, нанял адвоката, утверждения которого всегда признавались судом неоспоримой правдой.Следующий обмен произошел в суде.
Прокурор: «Если обвиняемый совершил преступление, у него был сообщник».
Защитник: «Это неправда!»
Адвокат помогал своему клиенту?

Проблемы логики в суде III

Вы живете на острове, где есть только два типа людей: те, кто всегда говорят правду (рассказчики правды), и те, кто всегда лжет (лжецы).Вы обвиняетесь в совершении преступления и предстаете перед судом, где вам разрешается высказать только одно предложение в свою защиту. Что вы скажете в каждой из следующих ситуаций?

  • Если бы вы были лжецом (суд этого не знает) и невиновны. И это установленный факт, что преступление совершил лжец.
  • Та же ситуация, что и выше, но преступление совершили вы.
  • Если бы вы говорили правду (суд этого не знает), и вы были бы невиновны.И это установленный факт, что преступление совершил правдивый.
  • Если вы были невиновны и это установленный факт, что преступление не было совершено «нормальным» человеком. Нормальные люди — это те новые иммигранты, которые иногда лгут, а иногда говорят правду. Какой приговор, независимо от того, были ли вы правдой, лжецом или нормальным человеком, может доказать вашу невиновность?

Ящик Пандоры I

Жила-была девушка по имени Пандора, которая хотела яркого жениха, поэтому придумала для подражателя несколько логических задач.Это одна из них.
Основываясь на надписях на коробках (нет или только одна из них верна), выберите ту коробку, в которой спрятано обручальное кольцо.

Золотая коробка
Кольцо находится в этой коробке.

Серебряная коробка
Кольца нет в этой коробке.

Свинцовый ящик
Кольца нет в золотой коробке.

Ящик Пандоры II

А вот и второй тест. По крайней мере одна надпись верна и по крайней мере одна ложная. Это означает, что кольцо находится в …

Золотая коробка
Кольца нет в серебряной шкатулке.

Серебряная коробка
Кольца нет в этой коробке.

Свинцовый ящик
Кольцо находится в этой коробке.

Лев и Единорог I

Алиса наткнулась на льва и единорога в лесу забвения. Эти двое — странные существа. Лев лжет каждый понедельник, вторник и среду, а в другие дни говорит правду. Единорог лежит по четвергам, пятницам и субботам, однако в другие дни недели он говорит правду.
Лев: Вчера солгал.
Единорог: Я тоже.
В какой день они это сказали?

Лев и Единорог II

Лев сказал: Вчера лгал, а через два дня снова буду лгать.
В какой день он это сказал?

Остров Ваал

На этом острове живут люди и странные обезьяны, и вы не можете сказать, кто есть кто.Они говорят либо только правду, либо только ложь.
Кто следующие два парня?
A: B — лежащая обезьяна. Я человек.
B: A говорит правду.

Истина, ложь и мудрость

Три богини сидели в старинном индийском храме. Их звали Истина (всегда говорит правду), Ложь (всегда ложь) и Мудрость (иногда ложь).Посетитель спросил того, что слева: «Кто с тобой сидит?»
«Правда», — ответила она.
Затем он спросил того, кто посередине: «Кто ты?»
«Мудрость.»
Наконец, он спросил того, кто справа: «Кто твой сосед?»
«Ложь», — ответила она.
И тогда стало понятно, кто есть кто.

В Альпах

Трое туристов спорят, куда им идти.Ганс говорит, что Эмануэль лжет. Эмануэль утверждает, что Ганс и Филипп говорят одинаково, только не знает, правда это или ложь.
Так кто точно врет?

Монеты

Представьте, что на столе лежат 3 монеты: золотая, серебряная и медная. Если вы сделаете правдивое заявление, вы получите одну монету. Если вы сделаете ложное заявление, вы ничего не получите.
Какой приговор может гарантировать вам получение золотой монеты?

Тонкий любовник

Что-нибудь, чтобы расслабиться.Стройный молодой человек пригласил девушку на свидание:
«Я говорю кое-что. Если это правда, вы дадите мне свою фотографию?»
«Да», — ответила мисс.
«И если это ложь, не давайте мне свою фотографию. Вы бы пообещали это?»
Девушка согласилась. Затем этот парень произнес такую ​​фразу, что, немного подумав, она поняла, что, если она хочет выполнить свое обещание, ей не нужно будет давать ему фотографию, а только поцелуй .
Что бы вы сказали (на его месте), чтобы вас поцеловали и так далее?

Поделитесь этой страницей с друзьями

31 сложные головоломки для бокового мышления (с ответами)

Боковое мышление определяется как решение проблем с помощью непрямого и творческого подхода, обычно путем рассмотрения проблемы в новом и необычном свете.Боковое мышление используется для перехода от известной идеи к созданию новых. Навык нестандартного мышления можно приобрести через практику и использование. Это дает ключ к новаторскому мышлению и творчеству.

В этой статье мы предлагаем несколько головоломок для нестандартного мышления, которые помогут вам начать мыслить нестандартно. Сначала мы зададим вам несколько вопросов о нестандартном мышлении. Затем мы предоставим вам несколько вопросов и ответы на вопросы о нестандартном мышлении, чтобы дать вам основу для вашего собственного нестандартного мышления.Наконец, мы насладимся забавными загадками для нестандартного мышления, которые пощекотят ваш разум и заставят мыслить нестандартно.

Вопросы для бокового мышления

Вопросы для бокового мышления создают сценарий, который при первом чтении кажется либо сбивающим с толку, либо невозможным. У них также может быть несколько решений, хотя есть одно, которое обычно считается «лучшим». Мы выбрали несколько хорошо известных вопросов о нестандартном мышлении, чтобы вы начали. Сначала мы перечислили вопросы, а затем список ответов.Много раз вы будете думать: «О, это было очевидно». когда представлен ответ. Мы также начали с тех, кого считаем самыми простыми, и прошли путь к самым сложным. Наслаждаться!

Вопросы для бокового мышления

Морковь, шарф и угольки

На лужайке лежат морковь, шарф и пять кусков угля. Никто не ставит их на лужайку, но есть простая и логичная причина, почему они там есть. Что это?

Адам и Ева на небесах

Человек умер и отправился на небеса, где были миллионы других людей.Все были обнажены и выглядели так же, как в 21 год. Он огляделся, чтобы узнать, узнал ли он кого-нибудь. Он увидел пару и сразу понял, что это Адам и Ева. Как он их узнал?

Double Sons

У женщины было два сына. Они родились в один и тот же час, в один и тот же день одного месяца одного года. Однако близнецами они не были. Как это могло произойти?

Мужчина и лифт

Мужчина жил на десятом этаже дома. Каждый день он спускался на лифте на первый этаж, чтобы пойти на работу или за покупками.Когда он вернулся, он поднялся на лифте на седьмой этаж, а затем поднялся по лестнице, чтобы добраться до своей квартиры на десятом этаже. В дождливый день он поднялся на лифте до десятого этажа. Он ненавидел ходить, так почему он это делает?

Повесившийся человек

В большом, совершенно пустом деревянном сарае мертвец висит на середине центрального стропила. Он висит на высоте трех футов от земли на веревке длиной десять футов. Ближайшая стена находится в 20 футах от повешенного.Забраться по стенам или по стропилам было бы невозможно. Под ним лужа воды. Как человек повесился?

Смертельное блюдо

Двое мужчин заказали обед в ресторане. Они оба заказали одни и те же блюда из меню. После того, как они оба его попробовали, один из мужчин вышел из ресторана и застрелился. Почему он так поступил?

Рука в посылке

Однажды мужчина получил посылку по почте. Внутри он обнаружил тщательно упакованную человеческую руку.Он осмотрел его, перепаковал и отправил другому человеку. Второй мужчина также внимательно осмотрел руку, затем отнес ее в лес и закопал. Почему они оба поступили так?

Ответы на вопросы для бокового мышления

Морковь, шарф и угольки

Из этих предметов дети строили снеговика, который теперь растаял.

Адам и Ева на Небесах

Адам и Ева были единственными людьми без пупка. Они не были рождены женщинами, поэтому у них никогда не было пуповины и, следовательно, у них никогда не было пупка.

Двойные сыновья

Два ее сына были частью пары тройняшек (или четверных и т. Д.).

Мужчина и лифт

Мужчина очень невысокого роста. Он может дотянуться только до кнопки седьмого этажа. Однако в дождливые дни он использует свой зонтик, чтобы нажать кнопку десятого этажа.

Человек, который повесился

Мужчина стоял на глыбе льда и зависал, когда он таял.

Смертельное блюдо

Еда, которую заказали двое мужчин, была рыба-меч.Много лет назад они оба оказались на необитаемом острове. Когда один из мужчин попробовал рыбу-меч, он понял, что никогда не пробовал ее раньше. Это означало, что мясо, которое ему дали на острове, не было меч-рыбой, как ему сказали. Он понял, что съел плоть своего сына, который умер, когда они впервые достигли острова. Эта мысль привела его к краю пропасти, и он покончил с собой.

Рука в посылке

Несколько лет назад трое мужчин застряли на необитаемом острове.Когда они отчаянно нуждались в еде, они согласились ампутировать левую руку, чтобы съесть ее. Один из них, врач, отрезал руки двум другим мужчинам. Поскольку они поклялись, что всем отрубят левую руку, после того, как их спасли, врач ампутировал руку и отправил его двум другим мужчинам.

Вопросы и ответы о боковом мышлении

В этом следующем разделе мы перечислили вопросы и ответы о боковом мышлении, не отделяя вопросы от ответа.Вы можете найти их значительно проще, чем в предыдущем разделе. Опять же, мы начали с тех, кто считает более легкими, и работали над более сложными. Возможно, вы обнаружите, что объединение вопросов и ответов поможет вам быстро и легко вспомнить перечисленные, чтобы вы могли поделиться ими с друзьями, семьей и коллегами.

Водитель автобуса

Вы ведете автобус. Автобус пуст, когда вы начинаете свой маршрут. На первую остановку садятся четыре человека. На второй остановке восемь человек садятся и двое выходят.На третьей остановке 3 человека выходят и 4 садятся. Какого цвета глаза водителя автобуса?

Вы ведете автобус, так какого цвета у вас глаза?

Тушеные куры

В одной семье был курятник для дюжины кур, дающих яйца. Однажды поздно ночью в их районе прокатился ужасный шторм, унесший жизни всех цыплят, кроме восьми. Сколько кур было в семье на следующее утро?
В семье оставалось двенадцать цыплят, четыре мертвых и восемь живых.

Украденные товары

Женщина, которая находится в универмаге, заполняет свою тележку для покупок доверху и выходит из магазина, не заплатив.Хотя ее видят, никто не вызывает полицию и не пытается ее остановить. Почему?
Женщина работает в универмаге. Она наполняет корзину мусором и выносит его в мусорный бак.

Ужасающий тигр

Жарким субботним днем ​​женщина медленно шла через саванну, когда заметила вдали тигра. Вместо того чтобы развернуться, спрятаться или обратиться за помощью, женщина побежала туда, где был тигр. Почему не испугалась и не убежала?

Женщина была в зоопарке.

Незнакомец в машине

Мужчина мчался на машине по улицам, а его жена была единственным пассажиром. Когда они остановились, муж вышел из машины. Когда он вернулся, он обнаружил свою жену мертвой и незнакомца в машине. Что произошло?
Жена была беременна, и они мчались в больницу для рождения ребенка. Муж выскочил из машины за помощью и инвалидной коляской. Когда он вернулся, его жена родила ребенка, но умерла при родах.

Мучение трубки

Женщина входит в большую металлическую трубку. Она охвачена страхом и крепко сжимает руку мужа. Она явно потрясена. Ее муж нежно потирает ее руки и мягко разговаривает с ней, но сам это не трогает. Через несколько часов муж женщины говорит ей, что пора уходить, и ее мучения заканчиваются. Что происходило с женщиной?

Женщина боится летать, но должна отправиться в путешествие, чтобы увидеть умирающего родственника; это добавляет ей эмоционального смятения.Ее муж делает все возможное, чтобы утешить ее в течение нескольких часов полета.

Умный папа

Отец Джейкоба не мог помешать своему сыну играть в видеоигры. Как только он перестанет смотреть на Джейкоба, он вернется к игре на выезде. Папа Джейкоба решил навсегда решить эту проблему. Он схватил свой молоток и решил проблему. Джейкоб больше не мог играть, однако отец Джейкоба мог это делать. Что сделал отец Джейкоба?
Отец Джейкоба использовал молоток, чтобы установить полку высоко на стене.Он поставил игровую приставку и контроллеры на полку вне досягаемости Джейкоба; однако отец Джейкоба все еще мог связаться с ними.

Выжил прыжок

Мужчина, живущий в пятидесятиэтажном здании, решает выпрыгнуть из окна. Он пережил падение без травм. Как он это сделал?

Мужчина выпрыгнул из окна на первом этаже, которое находилось всего в нескольких футах от земли.

Заблокировано

После долгой ночи вечеринок с друзьями, женщина приходит домой и обнаруживает, что она не может войти в дом.Она уверена, что оказалась в правильном доме, но не может попасть внутрь. Что случилось, что делает невозможным проникновение в ее дом?

У женщины ключи отобрал обеспокоенный друг, когда она начала опьянеть ранее той ночью. Женщина позвонила и наняла водителя, чтобы отвезти ее домой, но у нее не было ключей, чтобы отпереть двери.

Нарушение

У подножия большого холма более 40 автомобилей попали в аварию. Некоторые машины перевернуты, а другие лежат на других машинах.Накопление настолько велико, что в аварии также участвуют несколько военных машин и пожарная машина. Что стало причиной этого большого крушения?

Машины, грузовики и другие транспортные средства — это игрушки, в которые играет маленький ребенок.

Солнечный день Погода?

Мужчина за рулем в солнечный день. Он делает поворот, и вода внезапно начинает сильно забивать его машину примерно на пять минут. Затем машину бьет очень сильный ветер. Машину даже начинает трясти. Дул прекращается, мужчина делает еще один поворот на главную дорогу и благополучно направляется домой.Что происходит?

Мужчина на своей машине заехал на автомойку. Примерно через 10 минут он уезжает и едет домой на своей чистой машине.

Загадки для бокового мышления

Загадки для бокового мышления похожи на вопросы для бокового мышления. Однако формулировка немного другая, больше похожа на загадку. Мы выбрали несколько загадок нестандартного мышления, которые могут поставить в тупик даже самого умного мыслителя.

Выбор

Убийца приговорен к смерти.После вынесения приговора он должен выбрать одну из трех комнат. Первый полон бушующего огня. Во втором полно наемных убийц с заряженными ружьями. Третий полон львов, которые не ели полгода. Какую комнату ему выбрать?

Третья комната, потому что львы, которые не ели шесть месяцев, мертвы.

Смертельное решение

Мужчина решил открыть дверь. Он закричал и через несколько минут был найден мертвым. В этом районе не было слышно выстрелов и не было обнаружено оружия.Что случилось?

Мужчина находился в самолете. Он открыл люк и упал насмерть.

Пропавший без вести

Трое мужчин входят в комнату, но только двое выходят. Комната пуста. Где третий мужчина?

Третий мужчина использует инвалидную коляску, поэтому он выкатывается из комнаты, вместо того, чтобы выйти.

Толкать машину

Мужчина толкнул свою машину. Когда он добрался до гостиницы, он понял, что обанкротился. Откуда он это узнал и почему стал банкротом?

Мужчина играл в «Монополию».

Умный бармен

Мужчина вошел в бар и попросил у бармена стакан воды. Бармен достает пистолет и направляет его на мужчину. Мужчина говорит: «Спасибо» и уходит. Что на самом деле произошло?

Мужчина икнул. Когда бармен узнал это, он вытащил пистолет, чтобы потрясти мужчину. Это сработало, и мужская икота исчезла. Тогда человеку больше не нужна была вода.

Смерть и неоткрытый сверток

Мужчина найден мертвым в поле.Рядом с ним — неоткрытый пакет. С ним в поле нет ни одного человека или живого существа. Как он умер?

Мужчина выпрыгнул из самолета с не раскрытым парашютом. Парашют — это неоткрытая упаковка.

Классический метод решения проблем — это логическое или вертикальное мышление — прийти к выводу, проработав пошаговый процесс с использованием данной информации. Боковое или горизонтальное мышление предполагает наличие множества идей и поиск альтернативных способов решения проблемы путем рассмотрения информации в более широком смысле.Использование головоломок нестандартного мышления может помочь вам раскрыть свою способность подходить ко многим жизненным проблемам и вызовам по-новому и увлекательно. Мы надеемся, что эта статья начнет вас на пути к продуктивному нестандартному мышлению.

Добавить комментарий

Ваш адрес email не будет опубликован. Обязательные поля помечены *